You are on page 1of 353

Functions that Describe

Situations

Andrew Gloag
Melissa Kramer
Anne Gloag

Say Thanks to the Authors


Click http://www.ck12.org/saythanks
(No sign in required)
AUTHORS
Andrew Gloag
To access a customizable version of this book, as well as other
Melissa Kramer
interactive content, visit www.ck12.org
Anne Gloag

CK-12 Foundation is a non-profit organization with a mission to


reduce the cost of textbook materials for the K-12 market both in
the U.S. and worldwide. Using an open-source, collaborative, and
web-based compilation model, CK-12 pioneers and promotes the
creation and distribution of high-quality, adaptive online
textbooks that can be mixed, modified and printed (i.e., the
FlexBook® textbooks).

Copyright © 2016 CK-12 Foundation, www.ck12.org

The names “CK-12” and “CK12” and associated logos and the
terms “FlexBook®” and “FlexBook Platform®” ( collectively “CK-12
Marks”) are trademarks and service marks of CK-12 Foundation
and are protected by federal, state, and international laws.

Any form of reproduction of this book in any format or medium,


in whole or in sections must include the referral attribution link
http://www.ck12.org/saythanks (placed in a visible location) in
addition to the following terms.

Except as otherwise noted, all CK-12 Content (including CK-12


Curriculum Material) is made available to Users in accordance
with the Creative Commons Attribution-Non-Commercial 3.0
Unported (CC BY-NC 3.0) License (http://creativecommons.org/
licenses/by-nc/3.0/), as amended and updated by Creative
Commons from time to time (the “CC License”), which is
incorporated herein by this reference.

Complete terms can be found at http://www.ck12.org/about/ terms-


of-use.

Printed: June 9, 2016

www.ck12.org Chapter 1. Functions that Describe Situations


CHAPTER
1 Functions that Describe Situations
Here you’ll learn how to interpret situations that occur in everyday life and use functions to represent them. You’ll
also use these functions to answer questions that come up.
What if your bank charged a monthly fee of $15 for your checking account and also charged $0.10 for each check
written? How would you represent this scenario with a function? Also, what if you could only afford to spend $20
a month on fees? Could you use your function to find out how many checks you could write per month? In this
Concept, you’ll learn how to handle situations like these by using functions.

Guidance

Write a Function Rule

In many situations, data is collected by conducting a survey or an experiment. To visualize the data, it is arranged
into a table. Most often, a function rule is needed to predict additional values of the independent variable.

Example A

Write a function rule for the table.

Number of CDs 2 4 6 8 10
Cost ($) 24 48 72 96 120

Solution:

You pay $24 for 2 CDs, $48 for 4 CDs, and $120 for 10 CDs. That means that each CD costs $12.
We can write the function rule.
Cost = $12×number of CDs or f(x)= 12x

Example B

Write a function rule for the table.


1
www.ck12.org

x −3
−2 −1
0 1
2 3
y 3
2 1
0 1
2 3
Solution:

The values of the dependent variable are always the corresponding positive outcomes of the input values. This
relationship has a special name, the absolute value. The function rule looks like this: f(x)= |x|.
Represent a Real-World Situation with a Function.

Let’s look at a real-world situation that can be represented by a function.

Example C

Maya has an internet service that currently has a monthly access fee of $11.95 and a connection fee of $0.50 per
hour. Represent her monthly cost as a function of connection time.

Solution:

Let x = the number of hours Maya spends on the internet in one month, and let y = Maya’s monthly cost. The
monthly fee is $11.95 with an hourly charge of $0.50.
The total cost = flat fee + hourly fee × number of hours. The function is y = f(x)= 11.95+0.50x.

Guided Practice

When diving in the ocean, you must consider how much pressure you will experience from diving a certain depth.
From the atmosphere, we experience 14.7 pounds per square inch (psi) and for every foot we dive down into the
ocean, we experience another 0.44 psi in pressure.
a.) Write a function expressing how pressure changes depending on depth underwater.
b.) How far can you dive without experiencing more than 58.7 psi of pressure on your body?
Solution:

a.) We are always experiencing 14.7 psi from the atmosphere, and that pressure increases by 0.44 psi for every
foot we descend into the ocean. Let d be our depth in feet underwater. Our dependent variable is the pressure P,
which is a function of d:

P = P(d)= 14.7+0.44d
b.) We want to know what our depth would be for a pressure of 58.7 psi.

58.7 = 14.7+0.44d

2
www.ck12.org Chapter 1. Functions that Describe Situations

Simplifying our equation by subtracting 14.7 from each side:

44 = 0.44d

What should d be in order to satisfy this equation? It looks like d should be 100. Let’s check:

44 = 0.44(100)= 44

So we do not want to dive more than 100 feet, because then we would experience more than 58.7 psi of pressure.

Explore More

1. Use the following situation: Sheri is saving for her first car. She currently has $515.85 and is saving $62 each
week.

a. Write a function rule for the situation.


b. Can the domain be “all real numbers"? Explain your thinking.
c. How many weeks would it take Sheri to save $1,795.00?
2. Write a function rule for the table.

x 3 4 5 6 y 9 16 25 36

3. Write a function rule for the table.

hours 0 1 2 3 cost 15 20 25 30

4. Write a function rule for the table.

x 0 1 2 3 y 24 12 6 3

5. Write a function that represents the number of cuts you need to cut a ribbon in x number of pieces.
6. Solomon charges a $40 flat rate and $25 per hour to repair a leaky pipe. Write a function that represents
the total fee charged as a function of hours worked. How much does Solomon earn for a three-hour job?
7. Rochelle has invested $2500 in a jewelry-making kit. She makes bracelets that she can sell for $12.50 each.
How many bracelets does Rochelle need to make before she breaks even?

3
ION
SECT Rational Equations and
7.5 Inequalities in One Variable
“One person’s constant is another person’s variable.”
7.5 OBJECTIVES
–Susan Gerhart
1. Solve rational
equations in one
variable alge-
braically
2. Solve literal
equations involv-
Rational Equations
ing a rational ex- Applications of your work in algebra will often result in equations involving rational
pression expressions. Our objective in this section is to develop methods to find solutions for
3. Solve rational in- such equations.
equalities in one The usual technique for solving such equations is to multiply both sides of the
variable alge- equation by the lowest common denominator (LCD) of all the rational expressions ap-
braically pearing in the equation. The resulting equation will be cleared of fractions, and we can
then proceed to solve the equation as before. Example 1 illustrates the process.

Exa mple 1 Clearing Equations of Fractions


Solve.
2x x
!! "!! #13
3 5

The LCD for 3 and 5 is 15. Multiplying both sides of the equation by 15, we have

2x x
! 3 5 "
15 !! "!! #15 $ 13 Distribute 15 on the left.

2x x
15 $ !! "15 $ !! #15 $ 13
3 5
10 x "3x #195 Simplify. The equation is now
cleared of fractions.
13x #195
x #15

The solution set is {15}.

528
Section 7.5 ■ Rational Equations and Inequalities in One Variable 529

To check, substitute 15 in the original equation.

2 $ 15 15
!! "!! ! 13
3 5
10 "3 ! 13
13 #13 A true statement.

So 15 is the solution for the equation.

Caution
! Be Careful! A common mistake is to confuse an equation such as

2x x
!! "!! #13
3 5

and an expression such as

2x x
!! "!!
3 5

Let’s compare.

2x x
Equation: !! "!! #13
3 5

Here we want to solve the equation for x, as in Example 1. We multiply both sides by
the LCD to clear fractions and proceed as before.

2x x
Expression: !! "!!
3 5

Here we want to find a third fraction that is equivalent to the given expression. We
write each fraction as an equivalent fraction with the LCD as a common denominator.

2x x 2x $ 5 x$3
!! "!! #!! "!!
3 5 3$5 5$3
10x 3x 10x " 3x
#!! "!! #!!
15 15 15
13x
#!!
15


✓ CHECK YOURSELF 1
Solve.

3x x
!! % !! #7
2 3
530 Chapter 7 ■ Rational Expressions

The process is similar when variables are in the denominators. Consider Example 2.

Exa mple 2 Solving an Equation Involving Rational Expressions


Solve.

We assume that x cannot 7 3 1


have the value 0. Do you see
!! % !!2 #!!2
4x x 2x
why?
The LCD of 4x, x2, and 2x2 is 4x2. So, multiplying both sides by 4x2, we have

7 3 1
!
4x x "
4x2 !! % !!2 #4x2 $ !!2
2x
Distribute 4x2 on the left side.

7 3 1
4x2 $ !! % 4x2 $ !!2 #4x2 $ !! Simplify.
4x x 2x2
7x % 12 #2
7x #14
x #2

The solution set is {2}.

We leave the check of the solution, x #2, to you. Be sure to return to the original equa-
tion and substitute 2 for x.


✓ CHECK YOURSELF 2
Solve.

5 4 7
!! % !!2 #!!2
2x x 2x

Example 3 illustrates the same solution process when there are binomials in the
denominators.

Exa mple 3 Solving an Equation Involving Rational Expressions


Solve.

Here we assume that x 4 3x


cannot have the value %2 or
!! "3 #!!
x "2 x%3
3.
Section 7.5 ■ Rational Equations and Inequalities in One Variable 531

The LCD is (x "2)(x % 3). Multiplying by that LCD, we have

4 3x
Note that multiplying each
term by the LCD is the same
! "
(x "2)(x % 3) !! "(x "2)(x % 3)(3) #(x "2)(x % 3) !!
x "2 x%3 ! "
as multiplying both sides of
the equation by the LCD. Or, simplifying each term, we have

4(x % 3) "3(x "2)(x % 3) #3x(x "2)

We now clear the parentheses and proceed as before.

4x % 12 "3x2 % 3x % 18 #3x2 "6x


3x2 "x % 30 #3x2 "6x
x % 30 #6x
%5x #30
x #%6

The solution set is {%6}.

Again, we leave the check of this solution to you.


✓ CHECK YOURSELF 3
Solve.

5 2x
!! "2 #!!
x%4 x%3

Factoring plays an important role in solving equations containing rational


expressions.

Exa mple 4 Solving an Equation Involving Rational Expressions


Solve.

3 7 2
!! % !! #! !
x%3 x "3 x2 % 9

In factored form, the denominator on the right side is (x % 3)(x "3), which forms the
LCD, and we multiply each term by that LCD.

3 7 2
! " ! "
(x % 3)(x "3) !! % (x % 3)(x "3) !! #(x % 3)(x "3) !!
x%3 x "3 (x % 3)(x "3) # $
532 Chapter 7 ■ Rational Expressions

Again, simplifying each term on the right and left sides, we have

3(x "3) % 7(x % 3) #2


3x "9 % 7x "21 #2
%4x #%28
x #7

The solution set is {7}.

Be sure to check this result by substitution in the original equation.


✓ CHECK YOURSELF 4
4 3 5
Solve !! % !! #! !.
x%4 x "1 x2 % 3x % 4

Whenever we multiply both sides of an equation by an expression containing a


variable, there is the possibility that a proposed solution may make that multiplier 0.
As we pointed out earlier, multiplying by 0 does not give an equivalent equation, and
therefore verifying solutions by substitution serves not only as a check of our work but
also as a check for extraneous solutions. Consider Example 5.

Exa mple 5 Solving an Equation Involving Rational Expressions


Solve.

Note that we must assume x 2


!! % 7 #!!
that x & 2. x%2 x%2

The LCD is x % 2, and multiplying, we have

x 2
!!
x % 2"
! (x % 2) % 7(x % 2) # !!!"(x % 2)
Note that each of the three
terms gets multiplied by x%2
(x % 2).
Simplifying yields

x % 7(x % 2) #2
x % 7x "14 #2
%6x #%12
x #2
Section 7.5 ■ Rational Equations and Inequalities in One Variable 533

To check this result, by substituting 2 for x, we have

Caution
! 2 2
!! % 7 ! !!
Because division by 0 is 2%2 2%2
undefined, we conclude that
2 is not a solution for the 2 2
!! % 7 ! !!
original equation. It is an 0 0
extraneous solution. The
original equation has no The solution set is empty. The set is written {} or '.
solution.


✓ CHECK YOURSELF 5
x%3 1
Solve !! #4 "!!.
x%4 x%4

Equations involving rational expressions may also lead to quadratic equations, as


illustrated in Example 6.

Exa mple 6 Solving an Equation Involving Rational Expressions


Solve.

Assume x & 3 and x & 4. x 15 2x


!! #!! % ! !
x%4 x%3 x2 % 7x "12

After factoring the trinomial denominator on the right, the LCD of x % 3, x % 4,


and x2 % 7x "12 is (x % 3)(x % 4). Multiplying by that LCD, we have

x 15 2x
! " ! "
(x % 3)(x % 4) !! #(x % 3)(x % 4) !! % (x % 3)(x % 4) !!
x%4 x%3 (x % 3)(x % 4) # $
Simplifying yields

x(x % 3) #15(x % 4) % 2x Remove the parentheses.


2
x % 3x #15x % 60 % 2x Write in standard form and factor.
x2 % 16x "60 #0
(x % 6)(x % 10) #0

So

x #6 or x #10

Verify that 6 and 10 are both solutions for the original equation. The solution set is
{6, 10}.
534 Chapter 7 ■ Rational Expressions


✓ CHECK YOURSELF 6
3x 2 36
Solve !! % !! #! !.
x "2 x "3 x2 " 5x "6

The following algorithm summarizes our work in solving equations containing ra-
tional expressions.

Solving Equations Containing Rational Expressions


Step 1 Clear the equation of fractions by multiplying both sides of the equation
by the LCD of all the fractions that appear.
Step 2 Solve the equation resulting from step 1.
Step 3 Check all solutions by substitution in the original equation.

The method in this section may also be used to solve certain literal equations for
a specified variable. Consider Example 7.

Exa mple 7 Solving a Literal Equation


A parallel electric circuit. The R1
symbol for a resistor is

R2

Recall that the numbers 1 and If two resistors with resistances R1 and R2 are connected in parallel, the combined re-
2 are subscripts. We read R1 sistance R can be found from
as “R sub 1” and R2 as “R
sub 2.” 1 1 1
!! #!! "!!
R R1 R2

Solve the formula for R.


First, the LCD is RR1R2, and we multiply:

1 1 1
RR1R2 $ !! #RR1R2 $ !! "RR1R2 $ !!
R R1 R2
Section 7.5 ■ Rational Equations and Inequalities in One Variable 535

Simplifying yields

R1R2 #RR2 "RR1 Factor out R on the right.


R1R2 #R(R2 "R1) Divide by R2 "R1 to isolate R.
R R2 R R2
!1! #R or R #!1!
R2 " R1 R1 " R2


✓ CHECK YOURSELF 7
Solve for D1.

Note: This formula involves 1 1 1


!! #!! "!!
the focal length of a convex F D1 D2
lens.

The Distance Formula


d #r $ t Distance equals rate times time.

Sometimes use of this formula leads to a rational equation, as in Example 8.

Exa mple 8 Solving a Motion Problem


A boat can travel 16 miles per hour in still water. If the boat can travel 5 miles down-
stream in the same time it takes to travel 3 miles upstream, what is the rate of the river’s
current?
Step 1 We want to find the rate of the current.
Step 2 Let c be the rate of the current. Then 16 "c is the rate of the boat going
downstream and 16 % c is the rate going upstream.
Step 3 We’ll use a chart to help us set up an appropriate equation.

d r t

5
Downstream 5 miles 16 "c !!
16 "c
3
Upstream 3 miles 16 % c !!
16 % c
536 Chapter 7 ■ Rational Expressions

d
We found the time by dividing distance by rate. If d #r $ t, then t #!!. The key to
r
finding our equation is noting that the time is the same upstream and downstream, so

5 3
!! #!!
16 "c 16 % c

Step 4 Multiplying both sides by the LCD (16 "c)(16 % c) yields

5(16 % c) #3(16 "c)


80 % 5c #48 "3c
32 #8c
c #4

The current is moving at 4 miles per hour.

5 3
Step 5 To check, verify that !! #!!.
16 "4 16 % 4


✓ CHECK YOURSELF 8
A plane flew 540 miles into a steady 30 mi/h wind. The pilot then returned along the
same route with a tailwind. If the entire trip took seven and one-half hours, what would
his speed have been in still air?

Another application that frequently results in rational equations is the work prob-
lem. Example 9 illustrates.

Exa mple 9 Solving a Work Problem


One computer printer can print a company’s paychecks in 40 minutes (min). A second
printer can print them in 80 min. If both printers are working, how long will it take to
print the paychecks?

Before we learn how to solve work problems, we should look at a couple of common
errors made in attempting to solve such problems.

Error 1: Students sometimes try adding the two times together. If we add the 40

Caution
! min and the 80 min we get 120 min. Is this a reasonable answer? Certainly not!
If one printer does the job in 40 min, why would it take 120 min for two printers
to do it? It wouldn’t.
Section 7.5 ■ Rational Equations and Inequalities in One Variable 537

Error 2: A more reasonable approach would be to give half the job to each printer.

Caution
! The first printer would finish its half of the job in 20 min. The second would fin-
ish its half in 40 min. The first printer would be idle for the final 20 min, so we
know the job could have been finished faster.

Although error 2 does not solve the problem, it does give us guidelines for a reason-
able answer. When the printers work together, it will take them somewhere between
20 and 40 min to finish the job. To find the exact amount of time, we use the work
principle.
The Work Principle: Given an object, A, which completes a task in time a, and
an object, B, which completes the same task in time b, we find the amount of time
it takes them to complete the task together by solving the following equation for
variable t.

t t
!! "!! #1
a b

Now we can solve the problem.

Step 1 We are looking for the time it takes to print the paychecks.
Step 2 Our variable, t is the time it will take the printers to complete the task.
t t
Step 3 We have the equation !! "!! #1
40 80
Step 4 Multiply by LCD 80.

2t "t #80
3t #80
80
t #!! min
3
2
#26!! min
3
#26 min 40 sec

Step 5 To verify this answer, we find what fraction of the job each printer does in
80 80
!! !!
3 2 3 1
this time. The first printer does — #!! of the job. The second printer does — #!!
40 3 80 3
2 1
!
of the job. Together, they do the entire job !! "!! #1 .
3 3 "

✓ CHECK YOURSELF 9
It would take Sasha 48 days to paint the house. Natasha could do it in 36 days. How
long would it take them to paint the house if they worked together?
538 Chapter 7 ■ Rational Expressions

The techniques we have discussed can also be used to find the zeros of rational
functions. Remember that a zero of a function is a value of x for which f(x) #0.

Exa mple 10 Finding the Zeros of a Function


Find the zeros of

1 3 4
f(x) #!! % !! % !!
x 7x 21

Set the function equal to 0, and solve the resulting equation for x.

1 3 4
f(x) #!! % !! % !! #0
x 7x 21

The LCD for x, 7x, and 21 is 21x. Multiplying both sides by 21x, we have

1 3 4
!
x 7x 21 "
21x !! % !! % !! #21x $ 0

21 % 9 % 4x #0 Distribute 21x on the left side.


12 % 4x #0 Simplify.
12 #4x
3 #x

So 3 is the value of x for which f(x) #0, that is, 3 is a zero of f(x).


✓ CHECK YOURSELF 10
Find the zeros of the function.

5x " 2 11
f(x) #!! % !!
x%5 4

Rational Inequalities
To solve inequalities involving rational expressions, we need some properties of divi-
sion over the real numbers. Recall that

1. The quotient of two positive numbers is always positive.


2. The quotient of two negative numbers is always positive.
3. The quotient of a positive number and a negative number is always negative.

We solve rational inequalities by using sign graphs, as Example 11 illustrates.


Section 7.5 ■ Rational Equations and Inequalities in One Variable 539

Exa mple 11 Solving a Rational Inequality


Solve.

x%3
!! ( 0
x%3 x "2
The graph of y #!! is
x "2
y
The inequality states that the quotient of x % 3 and x "2 must be negative (less than
0). This means that the numerator and denominator must have opposite signs.
We start by finding the critical points. These are points where either the numera-
tor or denominator is 0. In this case, the critical points are 3 and %2.
The solution depends on determining whether the numerator and denominator are
x positive or negative. To visualize the process, start with a number line and label it as
shown below.

Sign of x % 3 % % % % % % % % % % % % " " " " "


Sign of x " 2 % % % " " " " " " " " " " " " " "

Sign of quotient " " " % % % % % % % % % " " " " "
For what values of x is y x%3 %2 3
less than 0? x "2

Examining the sign of the numerator and denominator, we see that

For any x less than %2, the quotient is positive (quotient of two negatives).
For any x between %2 and 3, the quotient is negative (quotient of a negative and
a positive).
For any x greater than 3, the quotient is positive (quotient of two positives).

We return to the original inequality

x%3
!! ( 0
x "2

This inequality is true only when the quotient is negative, that is, when x is between
%2 and 3. This solution set can be written as {x)%2 ( x ( 3} and represented on a
graph as follows.

%2 3
540 Chapter 7 ■ Rational Expressions


✓ CHECK YOURSELF 11
Solve and graph the solution set.

x%4
!! * 0
x "2

The solution process illustrated in Example 11 is valid only when the rational ex-
pression is isolated on one side of the inequality and is related to 0. If this is not the
case, we must write an equivalent inequality as the first step, as Example 12 illustrates.

Exa mple 12 Solving a Rational Inequality


Solve.

2x % 3
!! + 1
x "1

Since the rational expression is not related to 0, we use the following procedure.

2x % 3
We have subtracted 1 from !! % 1 + 0 Form a common denominator on the left side.
both sides. x "1
2x % 3 x "1
!! % !! + 0 Combine the expressions on the left side.
x "1 x "1
2x % 3 % (x "1)
!! + 0 Simplify.
x "1
x%4
!! + 0
x "1

We can now proceed as before since the rational expression is related to 0. The criti-
cal points are 4 and %1, and the sign graph is formed as shown below.

Sign of x % 4 % % % % % % % % % % % " " " "


Sign of x " 1 % % % " " " " " " " " " " " "

Sign of quotient " " " % % % % % % % % " " " "


x%4 %1 4
x "1
Section 7.5 ■ Rational Equations and Inequalities in One Variable 541

We want a positive quotient. From the sign graph the solution is

{x)x ( %1 or x + 4}

The graph is shown below.

Note that 4 is included, but [


%1 cannot be included in the %1 4
solution set. Why?


✓ CHECK YOURSELF 12
Solve and graph the solution set.

2x % 3
!! , 1
x%2


✓ CHECK YOURSELF ANSWERS

1. {6}. 2. {3}. 3. {9}. 4. {%11}. 5. No solution or '.


8 FD2 4
%
6. %5, !! .
3 & 7. !!.
D2 % F
8. 150 mi/h. 9. 20 !! days.
7
10. 7.

11. {x)x ( %2 or x * 4}
%2 4

12. {x)1 , x ( 2} [
1 2
Exercises ■ 7.5
1. Equation, {36} In Exercises 1 to 8, decide whether each of the following is an expression or an equa-
tion. If it is an equation, find a solution. If it is an expression, write it as a single
2. Equation, {28}
fraction.
3x
3. Expression, !!
10 x x x x x x
1. !! % !! #6 2. !! % !! #3 3. !! % !!
x 2 3 4 7 2 5
4. Expression, !!
24
x x 3x "1 3x % 1 x x "3
5. Equation, {5} 4. !! % !! 5. !! #x % 1 6. !! % !! % !!
6 8 4 2 5 4
21x % 25
6. Expression, !! x x 1 2x % 1 x
20 7. !! #!! "!! 8. !! "!!
4 12 2 3 2
7. Equation, {3}

7x % 2 In Exercises 9 to 50, solve each equation.


8. Expression, !!
6
9. {5} 10. {%7} x 3 x 7 x 1 x 1 4 3 10
9. !! "!! #!! "!! 10. !! % !! #!! "!! 11. !! "!! #!!
3 2 6 3 10 5 5 2 x 4 x
11. {8} 12. {6}
3 3 5 7 5 1 1 7 1 1
% &
13. !!
2
14. {2} 12. !! #!! % !!
x 3 x
13. !! % !! #!!
4x 2 2x
14. !! % !! #!!
6x 3 2x
15. {%1} 16. {%3} 3 2 5 4 9 2x
15. !! #!! 16. !! #!! 17. !! "2 #!!
9 2 x "4 x "3 x%2 x "1 x x "3
% &
17. %!!
5 % &
18. %!!
3
6 3x 3 5 13 7 2 6
2
18. !! "3 #!! 19. !! % !! #!! 20. !! % !! #!!
% &
19. %!!
3
20. {%3} x x "1 x "2 x x "2 x x%3 x

3 2 1 2 5 3
21. No solution or ' 21. !! "!! #!! 22. !! "!! #!!
2 2x % 4 x%2 x%1 2x % 2 4
22. {7} 23. {%23}
x x%1 5 x x%4 1
24. {5} 25. {6} 23. !! "!! #!! 24. !! % !! #!!
3x " 12 x "4 3 4x % 12 x%3 8
26. x is any real except
x #0 and x #2 x%1 x%3 3 x "1 x "3 6
25. !! % !! #! ! 26. !! % !! #! !
x "3 x x2 " 3x x%2 x x2 % 2x
27. {4} 28. {6}
1 2 2 1 1 12
29. {8} 30. {5} 27. !! % !! #! ! 28. !! "!! #! !
x%2 x "2 x2 % 4 x "4 x%4 x2 % 16
31. {4} 32. {%4}
7 1 x 2 3 x
29. !! % !! #! ! 30. !! #!! "! !
x "5 x%5 x2 % 25 x%2 x "2 x2 % 4

11 5 1 5 1 2
31. !! % ! ! #!! 32. !! #!! % ! !
x "2 x2 % x % 6 x%3 x%4 x "2 x2 % 2x % 8

542
Section 7.5 ■ Rational Equations and Inequalities in One Variable 543

3 11 5 3 24 3 5 2
% &
33. !!
2 % &
34. !!
2
33. !! % !! #!
x%2 x "3
!
x2 " x % 6
34. !! % !! #!
x "1 x "6
!
x2 "7x "6

35. No solution or ' x 3 x 5


35. !! % 2 #!! 36. !! "2 #!!
x%3 x%3 x%5 x%5
36. No solution or '

37. {7} 38. {3} 2 1 2 2 4 3


37. ! !%! ! #! ! 38. ! !%! ! #! !
39. {5} 40. {%3} x2 % 3x x2 " 2x x2 % x % 6 x2 % x x2 "5x % 6 x2 " 6x
5
% &
41. %!!
2
39. !
2
!%!
3 2
2 2 ! #!2 !
x % 4x "3 x %9 x "2x % 3
42. No solution or '

1
%
43. %!!, 6
2 & 40. !
2
!%!
x2 % 4
1
! #!
x2 " x % 2
3
!
x2 % 3x "2
44. {%4, 3}
7 3 40 3 18 5
1 2 41. !! % !! #! ! 42. !! % ! ! #!!
% &
45. %!!
2 % &
46. !!
3
x%5 x "5 x2 % 25 x%3 x2 % 9 x "3

1
%
47. %!!, 7
3 & 2x
x%3
2
43. !! "!! #!
x%5
3x
!
x2 % 8x "15
x
44. !! #!
x%4
5x
! % !!
x2 % x % 12
3
x "3
1
%
48. %!!, 6
2 & 2x
45. !! #!
5 1
! % !!
3x 2
46. !! #!! % !
2
!
2 2
x "2 x %x%6 x%3 x%1 x%2 x % 3x "2
49. {%8, 9}

9
%
50. %!!, 10
2 & 7 16
47. !! "!! #3
x%2 x "3
5 6
48. !! "!! #2
x%2 x "2
ab bx
51. !! 52. !!
b%a b%x
11 10 17 10
49. !! % 1 #!! 50. !! % 2 #!!
RR2 D F x%3 x "3 x%4 x "2
53. !! 54. !1!
R2 % R D1 % F

y "1 2y % 3 In Exercises 51 to 58, solve each equation for the indicated variable.
55. !! 56. !!
y%1 y%1

A IR
57. !! 58. !! 1 1 1 1 1 1 1 1 1
1 " rt E % Ir 51. !! #!! % !! for x 52. !! #!! "!! for a 53. !! #!! "!! for R1
x a b x a b R R1 R2

1 1 1 x "1 x%3
54. !! #!! "!! for D2 55. y #!! for x 56. y #!! for x
F D1 D2 x%1 x%2

A%P nE
57. t #!! for P 58. I #!! for n
Pr R " nr
544 Chapter 7 ■ Rational Expressions

59. {x)%1 ( x ( 2} In Exercises 59 to 72, solve each inequality, and graph the solution set.

x%2 x "3 x%4


60. {x)x ( %3 or x * 2} 59. !! ( 0 60. !! * 0 61. !! * 0
x "1 x%2 x%2

x "6 x%5 x "3


62. !! ( 0 63. !! , 0 64. !! , 0
61. {x)x ( 2 or x * 4} x%3 x "3 x%2

2x % 1 3x % 2 x 2
65. !! + 0 66. !! , 0 67. !! "!! , 0
62. {x)%6 ( x ( 3} x "3 x%4 x%3 x%3

x 3 x 4 x 2
68. !! % !! * 0 69. !! , !! 70. !! + !!
63. {x)%3 ( x , 5}
x "5 x "5 x "3 x "3 x%5 x%5

2x % 5 2x " 3
71. !! * 1 72. !! + 1
64. {x)%3 , x ( 2} x%2 x "4

In Exercises 73 to 80, find the zeros of each function.


1
%
65. x)x ( %3 or x + !!
2 &
x 12 4x x
73. f(x) #!! % !! 74. f(x) #!! % !!
10 5 3 6
2
%
66. x)!! , x ( 4
3 & 12 5 1 3
75. f(x) #!! % !! 76. f(x) #!! % !!
x "5 x x%2 x

67. {x)%2 , x ( 3}

68. {x)x ( %5 or x * 3}

69. {x)%3 ( x , 4}
72. {x)x ( %4 or x + 1}

70. {x)x , 2 or x * 5}
73. 24

74. 0

71. {x)x ( 2 or x * 3} 25
75. !!
7
76. 3
Section 7.5 ■ Rational Equations and Inequalities in One Variable 545

3 1 2 5 2 1 3
77. !! 77. f(x) #!! "!! % !! 78. f(x) #!! % !! % ! !
4 x%3 x 3x x x "1 x2 "x
78. 1 39 16 72
79. f(x) #1 "!! % !! 80. f(x) #x % !! "1
x2 x x
79. 3, 13

80. %9, 8 Solve the following problems.


81. 6, 24
81. Number analysis. One number is 4 times another number. The sum of the recip-
82. 4, 6 5
rocals of the numbers is !!. Find the two numbers.
83. 9
24
82. Number analysis. The sum of the reciprocals of two consecutive even integers is
84. 5 equal to 10 times the reciprocal of the product of those integers. Find the two
integers.
85. 4 mi/h
83. Number analysis. If the same number is subtracted from the numerator and deno-
86. 4 mi/h
11 1
minator of !!, the result is !!. Find that number.
87. 150 mi/h 15 3
88. 40 mi/h 8
84. Number analysis. The numerator of !! is multiplied by a number. That same num-
89. 36 min 9
ber is subtracted from the denominator, and the result is 10. What is that number?
90. 30 h
85. Motion. A motorboat can travel 20 mi/h in still water. If the boat can travel 3 mi
downstream on a river in the same time it takes to travel 2 mi upstream, what is
the rate of the river’s current?
86. Motion. Janet and Michael took a canoeing trip, traveling 6 mi upstream along a
river, against a 2 mi/h current. They then returned downstream to the starting point
of their trip. If their entire trip took 4 h, what was their rate in still water?
87. Motion. A plane flew 720 mi with a steady 30-mi/h tailwind. The pilot then re-
turned to the starting point, flying against the same wind. If the round-trip flight
took 10 h, what was the plane’s airspeed?
88. Motion. A small jet has an airspeed (the rate in still air) of 300 mi/h. During one
day’s flights, the pilot noted that the plane could fly 85 mi with a tailwind in the
same time it took to fly 65 mi against the same wind. What was the rate of the
wind?
89. Work. One computer printer can print a company’s weekly payroll checks in 60
min. A second printer would take 90 min to complete the job. How long would it
take the two printers, operating together, to print the checks?
90. Work. An electrician can wire a house in 20 h. If she works with an apprentice,
the same job can be completed in 12 h. How long would it take the apprentice,
working alone, to wire the house?
91. What special considerations must be made when an equation contains rational ex-
pressions with variables in the denominator?
x
92. In solving the inequality !! * 5, is it incorrect to find the solution by multi-
x%1
plying both sides by x % 1? Why? What technique should be used?
Key Concepts of Rational
Functions
Functions and their Graphs
Objectives

At the end of this lesson, the learner should be able to:


1. Find the domain and range of a rational function
2. Determine the intercepts, zeroes and asymptotes of rational functions
3. Graph rational functions
4. Solve problems involving rational functions, equations and inequalities
Finding the domain and range of
a rational function
Definitions

• Domain
• The set of all values of x for which the function is defined
• Range
• The set of all values of y for which the function takes from the
values of the domain
Determining the Domain and the Range

• When determining the domain and the range of a rational function, it is important
to note that the domain of the function should not include values of x which would
make the denominator zero (0)
• The domain of a rational function consists of all real number except for those
values which would make the denominator zero.
• One way to determine the range of a rational function is by getting the domain of
the inverse of the rational function
Example
• Get the domain and the range of 𝑓 𝑥 = 𝑥−1
𝑥+1

• Domain: all real number except 𝑥 = 1


• Getting the inverse of the function

• 𝑦=
𝑥+1
𝑥−1
𝑦 𝑥−1 =𝑥+1 𝑥𝑦 − 𝑦 = 𝑥 + 1 𝑥𝑦 − 𝑥 = 𝑦 + 1

• 𝑥 𝑦−1 =𝑦+1
𝑦+1
𝑥 = 𝑦−1 𝑡ℎ𝑒 𝑖𝑛𝑣𝑒𝑟𝑠𝑒 𝑖𝑠
𝑥+1
𝑥−1

• Range = domain of the inverse 𝑎𝑙𝑙 𝑟𝑒𝑎𝑙 𝑒𝑥𝑐𝑒𝑝𝑡 𝑦 = 1


Example
• Get the domain and the range of 𝑓 𝑥 =
𝑥+1
𝑥

• Domain: all real except 𝑥 = 0


• Getting the inverse of the function:

• 𝑦=
𝑥+1
𝑥
𝑥𝑦 = 𝑥 + 1 𝑥𝑦 − 𝑥 = 1 𝑥 𝑦−1 =1 𝑥=
1
𝑦−1

• 1
The inverse is 𝑦 = 𝑥−1
• Range is all values of 𝑦 𝑒𝑥𝑐𝑒𝑝𝑡 𝑦 = 1
Determining the intercepts, zeroes
and asymptotes of rational functions
Graph rational functions
www.hccfl.edu/media/723099/l4-1140-sec%205-3.pptx
Definitions

• Intercept
• An intercept, or x-intercept and y-intercept, is a point on the axis where the graph of the
function passes through.
• Asymptote
• An asymptote is a vertical or horizontal line in the Cartesian plane that a graph approaches to
but never passes through.
• Zeros
• Zeros of a function are x-intercepts
The Graph of a Rational Function
• General Steps to Graph a Rational Function
• 1) Factor the numerator and the denominator
• 2) State the domain and the location of any holes in the graph
• 3) Simplify the function to lowest terms
• 4) Find the y-intercept (x = 0) and the x-intercept(s) (y = 0)
• 5) Identify any existing asymptotes (vertical, horizontal, or oblique)
• 6) Identify any points intersecting a horizontal or oblique asymptote.
• 7) Use test points between the zeros and vertical asymptotes to locate the graph above or below
the x-axis
• 8) Analyze the behavior of the graph on each side of an asymptote
• 9) Sketch the graph
Example
• 𝑓 𝑥 =
𝑥2 +𝑥−12
𝑥2 −4
• 1) Factor the numerator and the denominator

• 𝑓 𝑥 =
(𝑥+4)(𝑥−3)
(𝑥+2)(𝑥−2)
• 2) State the domain and the location of any holes in the graph
• Domain: (−∞, −2) ∪ (−2, 2) ∪ (2, ∞)
• No holes
• 3) Simplify the function to lowest terms

• 𝑓 𝑥 =
(𝑥+4)(𝑥−3)
(𝑥+2)(𝑥−2)
General Steps to Graph a Rational Function

• 4) Find the y-intercept (x = 0) and the x-intercept(s) (y = 0)


• y-intercept (x = 0) x-intercept(s) (y = 0)

• (0+4)(0−3)
𝑓 0 = (0+2)(0−2) Use numerator factors

• 𝑓 0 =
−12
−4
=3 𝑥+4 =0 𝑥−3 =0
• (0, 3) 𝑥 = −4 𝑥=3
• (−4, 0) (3, 0)
General Steps to Graph a Rational Function
• 5) Identify any existing asymptotes (vertical, horizontal, or oblique
𝑥 2 +𝑥−12
• 𝑓 𝑥 = 𝑥 2 −4
(𝑥+4)(𝑥−3)
𝑓 𝑥 = (𝑥+2)(𝑥−2)

• Horiz. Or Oblique Asymptotes Vertical Asymptotes


• Examine the largest exponents Use denominator factors
• Same ∴ Horiz. - use coefficients 𝑥+2 =0 𝑥−2=0

• 𝑦=1
1
𝑥 = −2 𝑥=2
• 𝐻𝐴: 𝑦 = 1 𝑉𝐴: 𝑥 = −2 𝑎𝑛𝑑 𝑥 = 2
General Steps to Graph a Rational Function
• 6) Identify any points intersecting a horizontal or oblique asymptote.
𝑥 2 +𝑥−12
• 𝑦 = 1 𝑎𝑛𝑑 𝑓 𝑥 = 𝑥 2−4
𝑥 2 +𝑥−12
• 1=
𝑥 2−4
• 𝑥 2 − 4 = 𝑥 2 + 𝑥 − 12
• −4 = 𝑥 − 12
• 8=𝑥
• (8,1)
General Steps to Graph a Rational Function
• 7) Use test points between the zeros and vertical asymptotes to locate the graph above or
below the x-axis
𝑎𝑏𝑜𝑣𝑒 𝑏𝑒𝑙𝑜𝑤 𝑎𝑏𝑜𝑣𝑒

-4 -2 2 3

• 𝑓 𝑥 =
(𝑥+4)(𝑥−3)
(𝑥+2)(𝑥−2)
𝑓 −3 =
(+)(−)
(−)(−)
=−

• 𝑓 −5 =
(−5+4)(−5−3)
(−5+2)(−5−2)
𝑓 −3 = 𝑏𝑒𝑙𝑜𝑤

• 𝑓 −5 =
(−)(−)
(−)(−)
=+ 𝑓 0 =
(+)(−)
(+)(−)
=+
• 𝑓 −5 = 𝑎𝑏𝑜𝑣𝑒 𝑓 0 = 𝑎𝑏𝑜𝑣𝑒
General Steps to Graph a Rational Function
• 7) Use test points between the zeros and vertical asymptotes to locate the graph
above or below the x-axis
𝑎𝑏𝑜𝑣𝑒 𝑏𝑒𝑙𝑜𝑤 𝑎𝑏𝑜𝑣𝑒 𝑏𝑒𝑙𝑜𝑤 𝑎𝑏𝑜𝑣𝑒

-4 -2 2 3

𝑓 2.5 = 𝑏𝑒𝑙𝑜𝑤 𝑓 4 = 𝑎𝑏𝑜𝑣𝑒


General Steps to Graph a Rational Function
• 8) Analyze the behavior of the graph on each side of an asymptote
𝑎𝑏𝑜𝑣𝑒 𝑏𝑒𝑙𝑜𝑤 𝑎𝑏𝑜𝑣𝑒 𝑏𝑒𝑙𝑜𝑤 𝑎𝑏𝑜𝑣𝑒

-4 -2 2 3

𝑥 → −2− 𝑓(𝑥) → −∞

𝑥 → −2+ 𝑓(𝑥) → ∞
General Steps to Graph a Rational Function
• 8) Analyze the behavior of the graph on each side of an asymptote
𝑎𝑏𝑜𝑣𝑒 𝑏𝑒𝑙𝑜𝑤 𝑎𝑏𝑜𝑣𝑒 𝑏𝑒𝑙𝑜𝑤 𝑎𝑏𝑜𝑣𝑒

-4 -2 2 3

𝑥 → 2− 𝑓(𝑥) → ∞

𝑥 → 2+ 𝑓 𝑥 → −∞
9) Sketch the graph
Example

1) Factor the numerator and the denominator

2) State the domain and the location of any holes in the


graph
Domain: (−∞, −5) ∪ (−5, −3) ∪ (−3, ∞)
Hole in the graph at 𝑥 = −5
3) Simplify the function to lowest terms
General Steps to Graph a Rational Function
• 4) Find the y-intercept (x = 0) and the x-intercept(s) (y = 0)
y-intercept (x = 0) x-intercept(s) (y = 0)
Use numerator factors
𝑥−2 =0
𝑥=2
(2, 0)
General Steps to Graph a Rational Function

• 5) Identify any existing asymptotes (vertical, horizontal, or oblique


𝑥 2 +3𝑥−10
• 𝑓 𝑥 =
𝑥 2 +8𝑥+15
𝑓 𝑥 =
(𝑥−2)
(𝑥+3)

Horiz. Or Oblique Asymptotes Vertical Asymptotes


Examine the largest Use denominator factors
exponents 𝑥+3 =0
Same ∴ Horiz. - use coefficients
𝑥 = −3
𝑉𝐴: 𝑥 = −3
𝐻𝐴: 𝑦 = 1
General Steps to Graph a Rational Function
• 6) Identify any points intersecting a horizontal or oblique asymptote.

𝑥+3=𝑥−2
3 = −2
𝑙𝑜𝑠𝑡 𝑣𝑎𝑟𝑖𝑎𝑏𝑙𝑒
𝑛𝑜 𝑝𝑜𝑖𝑛𝑡𝑠 𝑜𝑓 𝑖𝑛𝑡𝑒𝑟𝑠𝑒𝑐𝑡𝑖𝑜𝑛 𝑜𝑛 𝑡ℎ𝑒 𝑎𝑠𝑦𝑚𝑝𝑡𝑜𝑡𝑒
General Steps to Graph a Rational Function
• 7) Use test points between the zeros and vertical asymptotes to locate the graph
above or below the x-axis
𝑎𝑏𝑜𝑣𝑒 𝑏𝑒𝑙𝑜𝑤 𝑎𝑏𝑜𝑣𝑒

-3 2

𝑓 0 = 𝑏𝑒𝑙𝑜𝑤

𝑓 −4 = 𝑎𝑏𝑜𝑣𝑒 𝑓 3 = 𝑎𝑏𝑜𝑣𝑒
General Steps to Graph a Rational Function
• 8) Analyze the behavior of the graph on each side of an asymptote

-3 2

• (𝑥−2)
𝑓 𝑥 = (𝑥+3)

• 𝑥 → −3− 𝑓(𝑥) →
(−)
(0− )
𝑓(𝑥) → ∞

• 𝑥 → −3 +
𝑓(𝑥) →
(−)
(0+ )
𝑓 𝑥 → −∞
9) Sketch the graph
Example

• 𝑓 𝑥 =
𝑥2 +3𝑥+2
𝑥−1
• 1) Factor the numerator and the denominator

• 𝑓 𝑥 =
(𝑥+2)(𝑥+1)
𝑥−1
• 2) State the domain and the location of any holes in the graph
• Domain: (−∞, 1) ∪ (1, ∞)
• No holes
• 3) Simplify the function to lowest terms

• 𝑓 𝑥 =
(𝑥+2)(𝑥+1)
(𝑥−1)
General Steps to Graph a Rational Function

• 4) Find the y-intercept (x = 0) and the x-intercept(s) (y = 0)


• y-intercept (x = 0) x-intercept(s) (y = 0)

• 𝑓 0 =
(0+2)(0+1)
(0−1)
Use numerator factors

• 𝑓 0 =
2
−1
= −2 𝑥+2 =0 𝑥+1 =0
• (0, −2) 𝑥 = −2 𝑥 = −1
• (−2, 0) (−1, 0)
General Steps to Graph a Rational Function
• 5) Identify any existing asymptotes (vertical, horizontal, or oblique
𝑥 2 +3𝑥+2
• 𝑓 𝑥 =
𝑥−1
𝑓 𝑥 =
(𝑥+2)(𝑥+1)
(𝑥−1)

• Horiz. or Oblique Asymptotes Vertical Asymptotes


• Examine the largest exponents Use denominator factors
• Oblique: Use long division 𝑥−1 =0
𝑥 +4 𝑥=1
2
𝑥−1 x 3x 2
−𝑥 2 −+𝑥 𝑉𝐴: 𝑥 = 1
4𝑥 +2
−4𝑥 −+4
0
O𝐴: 𝑦 = 𝑥 + 4
General Steps to Graph a Rational Function
• 6) Identify any points intersecting a horizontal or oblique asymptote.

• 𝑦 = 𝑥 + 4 𝑎𝑛𝑑 𝑓 𝑥 =
(𝑥+2)(𝑥+1)
𝑥−1

• 𝑥+4 =
(𝑥+2)(𝑥+1)
𝑥−1

• (𝑥 + 4)(𝑥 − 1) = (𝑥 + 2)(𝑥 + 1)
• 𝑥 2 + 3𝑥 − 4 = 𝑥 2 + 3𝑥 + 2
• 𝑙𝑜𝑠𝑡 𝑣𝑎𝑟𝑖𝑎𝑏𝑙𝑒
• 𝑛𝑜 𝑝𝑜𝑖𝑛𝑡𝑠 𝑜𝑓 𝑖𝑛𝑡𝑒𝑟𝑠𝑒𝑐𝑡𝑖𝑜𝑛 𝑜𝑛 𝑡ℎ𝑒 𝑎𝑠𝑦𝑚𝑝𝑡𝑜𝑡𝑒
General Steps to Graph a Rational Function
• 7) Use test points between the zeros and vertical asymptotes to locate the graph above or
below the x-axis
𝑏𝑒𝑙𝑜𝑤 𝑎𝑏𝑜𝑣𝑒 𝑏𝑒𝑙𝑜𝑤 𝑎𝑏𝑜𝑣𝑒

-2 -1 1

• 𝑓 𝑥 =
(𝑥+2)(𝑥+1)
(𝑥−1)
𝑓 −1.5 =
(+)(−)
(−)
=+

• 𝑓 −4 =
(−)(−)
(−)
=− 𝑓 −1.5 = 𝑎𝑏𝑜𝑣𝑒 𝑓 3 =
(+)(+)
(+)
=+

• 𝑓 −4 = 𝑏𝑒𝑙𝑜𝑤 𝑓 0 =
(+)(+)
(−)
=− 𝑓 3 = 𝑎𝑏𝑜𝑣𝑒
• 𝑓 0 = 𝑏𝑒𝑙𝑜𝑤
General Steps to Graph a Rational Function
• 8) Analyze the behavior of the graph on each side of an asymptote

• 𝑓 𝑥 =
(𝑥+2)(𝑥+1)
(𝑥−1)

• 𝑥 → 1− 𝑓(𝑥) →
(+)(+)
(0− )
𝑓 𝑥 → −∞

• 𝑥→1 +
𝑓(𝑥) →
(+)(+)
(0+ )
𝑓 𝑥 →∞
9) Sketch the graph
Solve problems involving rational
functions, equations and inequalities
http://iws.collin.edu/jmerrill/1314/1314%20ppt_files/3.5-RationalFunctions.ppt
The Average Cost of Producing a Wheelchair

• A company that manufactures wheelchairs has costs given by the function


C(x) = 400x + 500,000, where the x is the number of wheelchairs produced per
month and C(x) is measured in dollars. The average cost per wheelchair for the
company is given by …
Original: C(x) = 400x + 500,000

• C(x) = 400x + 500,000


x
a. Find the interpret C(1000), C(10,000), C(100,000).
C(1000) = 900; the average cost of producing 1000 wheelchairs per month is
$900.
C(x) = 400x + 500,000
x

Find the interpret C(10,000)


C(10,000) = 450; the average cost of producing 10,000 wheelchairs per month is
$450.
Find the interpret C(100,000)
C(100,000) = 405; the average cost of producing 100,000 wheelchairs per month
is $405.
C(x) = 400x + 500,000
x

b. What is the horizontal asymptote for the average cost function?

• Since n = d (in degree) then y = 400

• Describe what this represents for the company.


C(x) = 400x + 500,000
x
The horizontal asymptote means that the more wheelchairs produced per month,
the closer the average cost comes to $400. Lower prices take place with higher
production levels, posing potential problems for small businesses.
Solving Rational Equations

• To solve a rational equation first multiply each side by the LCD.

• You must check for extraneous solutions. (A solution that doesn’t work)
SOLVE: LCD?
( x 3)(x 3)

x x 2
( x 3)(x 3) 2 ( x 3)(x 3)
x 3 x 3 ( xx 3)(9x 3)
Multiply by LCD

SOLVE x( x 3) x( x 3) 2

x 2 3x x 2 3x 2
Check for Extraneous solutions.
2x2 2 (plug answer into original problem
1 1 2 1 1 2
x2 1 1 3 1 3 12 9 1 3 1 3 ( 1) 2 9
x 1 1 1 2 1 1 2
2 4 8 4 2 8
1 1 1 1
4 4 4 4
SOLVE: LCD?
( x 2)(x 1)
x 1 2x x 1 ( x 2)(x 1)
( x 2)(x 1)
( xx 2 2)(3x 12) x 2 x 1

x 1 2 x( x 1) ( x 1)(x 2)
x 1 2x2 2x x2 x 2
2 x 2 3x 1 x2 x 2
x2 2x 1 0
CHECK ??
( x 1)(x 1) 0 -1 does not work so
there is NO SOLUTION
x 1
How to solve using the CALC.

• Set equal to zero and graph (have to use parenthesis.


• Y2 = 0
• Find where the graph crosses x-axis (2nd trace intersect).
2 x 2 2 x 2 x x 2
1
x 2 x 2 x 2
x x x 1 x 3 x 3 x2 9

Answer: X=0 Answer: X= -2 Answer: X= 1, -1


Word Problems
• The function below gives the concentration of the saline solution after adding x
milliliters of 0.5% solution to 100 milliliters of 2% solution.

100(0.02) x(0.005)
y
100 x
• How man ML of the 0.5% solution must be added to have a combined
concentration of 0.9%?
100(0.02) x(0.005)
.009
100 x
Answer: (graph? solve X=275
by hand)
• You earn a 75% on the first test of the quarter how
many consecutive 100% test scores do you need to
bring your test average up to a 95%?
Write a rational function. Answer: 75 100x
f ( x)
Find when the rational x 1
function will be 95%

75 100 x
95
x 1
To find the average of Answer: (graph) You will need to make
something: add and divide 100% on the next 4 test to
by total number bring your test average
up to a 95%
Distance = rate x time

When a problem involves “how fast”, “how far”, or “for how long”, think about distance
equation d =rt
1850 480(t )
• A plane flies 1850 miles at a speed of 480 mph. Find the time of the trip to the nearest hundredth.
Answer: t 3.85

• On the return trip the plane travels at the same speed (480) but a tail wind helps the plane move faster. The
total flying time for the round trip is 7.55 hours. Find the speed x of the tail wind.

1850 (480 x)(3.7)


Answer: x 20
Work Problems
Joe can paint a room in 8 hours and Steve can paint the same
room in 12 hours. How long will it take the two to paint the room
if they work together?

• “you have to think of the problem in terms of how much each person or thing does
in a given amount of time” Working together Mary and Sally
How much of the job can Joe do in can paint a fence in 5.14 hours.
one hour? Steve? Working alone Mary can paint the
fence in 9 hours. How long would it
1 1 1 take Sally to paint the fence alone?
8 12 t 1 1 1
5 1 24 9 x 5.14
t
24 t 5 x 11.98
A plane leaves Chicago and flies to San Rate * time = distance
Francisco (1850 miles away) with a headwind r *t d Time of a trip equals
the distance over
The same plane returns to Chicago with a speed d
tailwind. The round trip took 7.75 hours.If the 1850 t
t r
airplane cruises at 480 mph, what is the speed 480 x
of the wind? (Assume the winds are constant.)

X=wind speed
San Francisco
Chicago
1850 1850
7.75
480 x 480 x
1850
t
Solve by Graphing 480 x
Zoom fit, x-max to 50
X = 35.195 Write an equation
Key Concepts of Inverse Functions,
Exponential Functions and
Logarithmic Functions - Part 001
Objectives

At the end of this lesson, the learner should be able to:


1. Represent real-life situations using one-to one functions
2. Determine the inverse of a one-to-one function
3. Represent an inverse function through its (a) table of values, and (b) graph
4. Find the domain and range of an inverse function
Represent real-life situations
using one-to one functions
Definitions

• Domain
• The set of all values of x for which the function is defined
• Range
• The set of all values of y for which the function takes from the
values of the domain
One-to-One function

•Def: A function is a one-to-one function if


no two different ordered pairs have the same
second coordinate.
One-to-One Function

• A function f is one-to one if for and and b in its domain, f(a) = f(b) implies a = b.
• Other – each component of the range is unique.
Horizontal Line Test
A test for one-to one

•If a horizontal line intersects the graph of the


function in more than one point, the function
is not one-to one
Example

Image taken from http://www.math.toronto.edu/preparing-for-calculus/4_functions/we_3_one_to_one.html


Example: Circumference of a Circle

• The circumference function for a circle is a one-to-one function as no two values


for diameter (d) would give the same value for the circumference
• 𝐶 = 𝜋𝑑
Example: Price you pay for goods you buy

• If you are buying apples at Php 28.00 per piece, the function that represents the
amount you need to pay when you buy x number of apples is
• 𝑃𝑟𝑖𝑐𝑒 𝑡𝑜 𝑃𝑎𝑦 = 𝑃ℎ𝑝 28.00 (𝑥) where x is the number of apples you will buy
• This is one-to-one since no two values of x will give you the same amount you need
to pay.
Determine the inverse of a one-to-one function
Represent an inverse function through its (a)
table of values, and (b) graph
http://college.cengage.com/mathematics/shared/content/digital_lessons/inverse_functions.ppt
Existence of an Inverse Function

•A function f has an inverse function if and


only if f is one to one.
Find an Inverse Function

• 1. Determine if f has an inverse function using horizontal line test.


• 2. Replace f(x) with y
• 3. Interchange x and y
• 4. Solve for y
• 5. Replace y with
1
f ( x)
Definition of Inverse Function

• Let f and g be two functions such that f(g(x))=x for every x in the domain
of g and g(f(x))=x for every x in the domain of.
• g is the inverse function of the function f
Definition

• A function y = f (x) with domain D is one-to-one on D


if and only if for every x1 and x2 in D, f(x1) = f (x2) implies that x1 = x2.
• A function is a mapping from its domain to its range
so that each element, x, of the domain is mapped to one, and only one, element, f (x),
of the range.
• A function is one-to-one if each element f(x) of the
range is mapped from one, and only one, element, x,
of the domain.
Horizontal Line Test
y
(0, 7) (4, 7)
• Horizontal Line Test
y=7
• A function y = f (x) is one-to-one if and only if
no horizontal line intersects the graph of y = f (x) 2

in more than one point. x


2
• Example: The function
y = x2 – 4x + 7 is not one-to-one on the real numbers because the line y = 7
intersects the graph at both (0, 7) and (4, 7).
Example
• Example: Apply the horizontal line test to the graphs below to determine if the
functions are one-to-one. 3 2
a) y = x3 b) y = x + 3x – x – 1
y y
8
8

4
4
-4 4
-4 4
x x

one-to-one not one-to-one


Inverse Functions
Function y = |x| + 1 Inverse relation x = |y| + 1
x y x y
2 2
1 3 1 3
0 2 0 2
-1 1 -1 1
-2 -2
Domain Range Range Domain
Every function y = f (x) has an inverse relation x = f (y).
The ordered pairs of :
y = |x| + 1 are {(-2, 3), (-1, 2), (0, 1), (1, 2), (2, 3)}.
x = |y| + 1 are {(3, -2), (2, -1), (1, 0), (2, 1), (3, 2)}.
The inverse relation is not a function. It pairs 2 to both -1 and +1.
Ordered Pairs

• The ordered pairs of the function f are reversed to produce the ordered pairs of the
inverse relation.
• Example: Given the function
f = {(1, 1), (2, 3), (3, 1), (4, 2)}, its domain is {1, 2, 3, 4} and its range is {1, 2, 3}.
• The inverse relation of f is {(1, 1), (3, 2), (1, 3), (2, 4)}.
• The domain of the inverse relation is the range of the original function.
• The range of the inverse relation is the domain of the original function.
• The graphs of a relation and its inverse are reflections in the line y = x.
• Example: Find the graph of the inverse relation geometrically from the graph of f (x)
= y y=x
3
(x 2)
2
4
• The ordered pairs of f are 3
given by the equation y ( x 2) . x
4 -2 2

• The ordered pairs of the inverse are given by (y 3


2)
-2
( x 3 2)
( y 3 2) x y
x 4 4
4
Inverse Relation: Algebraically
• Example: Find the inverse relation algebraically for the
function f (x) = 3x + 2.
y = 3x + 2 Original equation defining f
x = 3y + 2 Switch x and y.
3y + 2 = x Reverse sides of the equation.
( x 2)
y= Solve for y.
3
To calculate a value for the inverse of f, subtract 2, then
divide by 3.
Inverse Function

• For a function y = f (x), the inverse relation of f


is a function if and only if f is one-to-one.
• For a function y = f (x), the inverse relation of f
is a function if and only if the graph of f passes the horizontal line test.
• If f is one-to-one, the inverse relation of f
is a function called the inverse function of f.
• The inverse function of y = f (x) is written y = f -1(x).
Example

• Example: From the graph of the function y = f (x), determine if the inverse relation
is a function and, if it is, sketch its graph.
y y = f -1(x)
• The graph of f passes the horizontal line test. y=x
• The inverse y = f(x)
relation is a function.
x
• Reflect the graph of f in the line y = x to
produce the graph of f -1.
Composition of a Function

• The inverse function is an “inverse” with respect to the operation of composition of


functions.
• The inverse function “undoes” the function,
that is, f -1( f (x)) = x.
• The function is the inverse of its inverse function,
that is, f ( f -1(x)) = x.
• Example: The inverse of f(x) = x3 is f -1(x) = 3
x .
3 3
f -1( f(x)) = x = x and f ( f -1(x)) = (3 x )3 = x.
Composition of Functions
x 1
Example: Verify that the function g(x) =
is the inverse of f(x) = 2x – 1. 2
( f ( x) 1) ((2 x 1) 1) 2x
g( f(x)) = = = =x
2 2 2
x 1
f(g(x)) = 2g(x) – 1 = 2( ) – 1 = (x + 1) – 1 = x
2
It follows that g = f -1.
Find the domain and range of an
inverse function
Inverse Function

• g is the inverse of f if the domains and ranges are interchanged.


• f = {(1,2),(3,4), (5,6)}
• g= {(2,1), (4,3),(6,5)}
1
g ( x) f ( x)
Inverse of a function

f 1, 2 , 3, 4 , 5,6

1
f 2,1 4,3 , 6,5
Inverse of function

f 1, 2 , 3, 2 , 5, 2

1
f 2,1 , 2,3 , 2,5
Key Concepts of Inverse Functions,
Exponential Functions and
Logarithmic Functions - Part 002
Objectives

At the end of this lesson, the learner should be able to:


1. Graph inverse functions
2. Solve problems involving inverse functions
3. Represent real-life situations using exponential functions
4. Distinguish between exponential function, exponential equation, and
exponential inequality
Graph of Inverse Functions
Solve problems involving inverse
functions
www.nutleyschools.org/.../PPT%203.3%20Graphs%20of%20Inverse%20Functions.ppt
Remember we talked about functions - taking a set X
and mapping into a Set Y

11
22
2
2
• An inverse function
4
33 4 would reverse that
6 process and map from
6
4 8 SetY back into Set X
4 8
55 1010

Set X Set Y
If we map what we get out of the function back, we
won’t always have a function going back!!!

1
2
2
4
3
6
4 8
5
Recall that to determine by the graph if an equation is a
function, we have the vertical line test.
• If a vertical line intersects the graph of an equation more than one time, the
equation graphed is NOT a function.

This is NOT a This is a function


This is a function function
If the inverse is a function, each y value could only be
paired with one x. Let’s look at a couple of graphs.

• Horizontal
Line Test to
see if the
inverse is a
function.
Look at a y value (for For any y value, a horizontal
example y = 3)and see if line will only intersection the
there is only one x value graph once so will only have
on the graph for it. one x value
• If a horizontal line intersects the graph of an equation more than one time, the
equation graphed is NOT a one-to-one function and will NOT have an inverse
function.

This is NOT a
This is a one-to-one This is NOT a
one-to-one function one-to-one
function function
Horizontal Line Test

• Used to determine whether a function’s inverse will be a function by seeing if the


original function passes the horizontal line test.
• If the original function passes the horizontal line test, then its inverse is a function.
• If the original function does not pass the horizontal line test, then its inverse is not
a function.
Steps for Finding the Inverse of a One-to-One Function

y = f -1(x)

Solve for y

Trade x and y
places
Replace f(x) with y

4
f x
2 x
Find the inverse of a function
• Example 1: y = 6x - 12

Step 1: Switch x and y: x = 6y - 12


Step 2: Solve for y:
Consider the graph of the function : f ( x ) 2 x 4

y 2x 4

x 4
y
2

1 x 4
The inverse function is f ( x)
2
Consider the graph of the function f ( x) 2x 4

An inverse function
y 2x 4 x ( 0, 4 ) is just a
rearrangement with
( 4, 0 ) x and y swapped.
x
( 3, 2)x x 4
y So the graphs
x 2 just swap x and
( 2, 3)
y!

1 x 4
The inverse function is f ( x)
2
Graph f(x) and f -1(x) on the same graph.
Graph f(x) and f -1(x) on the same graph.
Represent real-life situations
using exponential functions
https://people.richland.edu/jodell/
Ratios and Rates

Ratio is the quotient of two numbers or two quantities.


The ratio of the numbers
a
a and b can also be
written as a:b, or b .
The units associated with the ratio are
important.
The units should match.
If the units do not match, it is called a rate,
rather than a ratio.
Proportions

Proportion is two ratios (or rates) that are equal to each other.
a c
b d
We can rewrite the proportion by multiplying
by the LCD, bd.
This simplifies the proportion to ad = bc.
This is commonly referred to as the cross product.
Solving Proportions
Example
x 1 5
Solve the proportion for x.
x 2 3
3x 1 5x 2
3x 3 5x 10
2x 7
x 7
2
Solving Proportions
Example Continued
Substitute the value for x into the original
equation, to check the solution.
7 1
2 5
7 2 3
2
5
2 5
3 3
true
2
So the solution is x 7
2
Solving Proportions
Example
If a 170-pound person weighs approximately 65 pounds on Mars, how
much does a 9000-pound satellite weigh?

170 - pound person on Earth 65 - pound person on Mars


9000 - pound satellite on Earth x - pound satellite on Mars

170x 9000 65 585,000


x 585000 / 170 3441 pounds
Solving Proportions
Example

Given the following prices charged for various sizes of picante sauce, find the best
buy.
• 10 ounces for $0.99
• 16 ounces for $1.69
• 30 ounces for $3.29
Solving Proportions
Example Continued
Size Price Unit Price

10 ounces $0.99 $0.99/10 = $0.099

16 ounces $1.69 $1.69/16 = $0.105625

30 ounces $3.29 $3.29/30 $0.10967

The 10 ounce size has the lower unit price, so it is the


best buy.
Similar Triangles

In similar triangles, the measures of corresponding angles are equal, and


corresponding sides are in proportion.

Given information about two similar triangles, you can often set up a proportion that
will allow you to solve for the missing lengths of sides.
Similar Triangles
Example

Given the following triangles, find the unknown length y.

12 m
5m

10 m y
Similar Triangles
Example
1.) Understand

Read and reread the problem. We look for the corresponding sides in the 2
triangles. Then set up a proportion that relates the unknown side, as well.

2.) Translate

By setting up a proportion relating lengths of corresponding sides of the two


triangles, we get 12 10
5 y
Similar Triangles
Example continued
3.) Solve
12 10
5 y
12 y 5 10 50
y 50 25 meters
12 6
Similar Triangles
Example continued
4.) Interpret
Check: We substitute the value we found from
the proportion calculation back into the
problem.
12 10 60
true
5 25 25
6
State: The missing length of the triangle 25 meters
6
is
Finding an Unknown Number
Example
The quotient of a number and 9 times its reciprocal
is 1. Find the number.
1.) Understand

Read and reread the problem. If we let


n = the number, then
= the reciprocal of the number
1
n
Finding an Unknown Number
Example continued
2.) Translate

The quotient is
of
a number and 9 times its reciprocal 1

1
n 9 = 1
n
Finding an Unknown Number
Example continued
3.) Solve
1
n 9 1
n
9
n 1
n
n
n 1
9
n2 9
n 3, 3
Finding an Unknown Number
Example continued
4.) Interpret
Check: We substitute the values we found from the
equation back into the problem. Note that nothing
in the problem indicates that we are restricted to
positive values.
1 1
3 9 1 3 9 1
3 3
3 3 1 true 3 3 1 true
State: The missing number is 3 or –3.
Solving a Work Problem
Example
An experienced roofer can roof a house in 26 hours. A
beginner needs 39 hours to do the same job. How long will it
take if the two roofers work together?
1.) Understand
Read and reread the problem. By using the times for each roofer to complete the
job alone, we can figure out their corresponding work rates in portion of the job
done per hour.

Time in hrs Portion job/hr


Experienced roofer 26 1/26
Beginner roofer 39 /39
Together t 1/t
Solving a Work Problem
Example continued
2.) Translate
Since the rate of the two roofers working together
would be equal to the sum of the rates of the two
roofers working independently,

1 1 1
26 39 t
Solving a Work Problem
Example continued
3.) Solve

1 1 1
26 39 t
1 1 1
78t 78t
26 39 t
3t 2t 78
5t 78

t 78 / 5 or 15.6 hours
Solving a Work Problem
Example continued
4.) Interpret
Check: We substitute the value we found from the
proportion calculation back into the problem.
1 1 1
26 39 78
5
3 2 5
true
78 78 78
State: The roofers would take 15.6 hours working
together to finish the job.
Solving a Rate Problem
Example
The speed of Lazy River’s current is 5 mph. A boat travels 20
miles downstream in the same time as traveling 10 miles
upstream. Find the speed of the boat in still water.
1.) Understand
Read and reread the problem. By using the formula d=rt, we can rewrite the
formula to find that t = d/r.
We note that the rate of the boat downstream would be the rate in still water + the
water current and the rate of the boat upstream would be the rate in still water –
the water current.
Distance rate time = d/r
Down 20 r + 5 20/(r + 5)
Up 10 r – 5 10/(r – 5)
Solving a Rate Problem
Example continued
2.) Translate
Since the problem states that the time to travel
downstairs was the same as the time to travel
upstairs, we get the equation
20 10
r 5 r 5
Solving a Rate Problem
Example continued
3.) Solve
20 10
r 5 r 5
20 10
r 5 r 5 r 5 r 5
r 5 r 5
20 r 5 10 r 5
20r 100 10r 50
10r 150 r 15 mph
Solving a Rate Problem
Example continued
4.) Interpret
Check: We substitute the value we found from the
proportion calculation back into the problem.
20 10
15 5 15 5
20 10
true
20 10
State: The speed of the boat in still water is 15 mph.
Key Concepts of Inverse Functions,
Exponential Functions and
Logarithmic Functions - Part 002
Objectives

At the end of this lesson, the learner should be able to:


1. Solve exponential equations and inequalities
2. Represent an exponential function through its: (a) table of values, (b) graph, and
(c) equation
3. Find the domain and range of an exponential function
4. Determine the intercepts, zeroes, and asymptotes of an exponential function
Solve exponential equations and
inequalities
www.brookwoodhighschool.net/Portals/0/teachers/.../4.6 Notes (Purple).ppt
Vocabulary

• Exponential Equations: equations in which variable expressions occur as


exponents.
• Exponential inequalities in one variable: inequalities that can be written in the
form:
• abx + k < 0
• abx + k < 0
• abx + k > 0
• abx + k > 0
Example 1:
• Solve 125x = 25x + 1
• We want 125 and 25 to have the same base!
Example 2:

x=1
• Solve 4x = (1/2)x – 3 by graphing
You Try:
• Solve 3x = (1/9)x – 3

• Solve 43x = 2x + 1 by graphing


Example 3:
• Solve 43z < 8z + 1 Remember: when you
multiply or divide by a
• Just like an example 1, we want them to have the same base!
negative it flips the
inequality sign!
Example 4:

x > -1.5
• Solve 22x + 1 – 1 > -0.75 by graphing
You Try:
• Solve 42x – 5 < 643x

• Solve 24x – 3 + 0.75 > 1 by graphing


Represent an exponential function through its: (a)
table of values, (b) graph, and (c) equation
Find the domain and range of an exponential
function
bento.cdn.pbs.org/hostedbento-prod/filer.../ems/.../11_6_exponential_functions.ppt
Graphs of Exponential Functions
• Exponential function – A function of the form y=abx, where b>0 and b 1.
x
Step 1 – Make a table of values for the function. y 3
x y 3x y
2 1 1
2 3
32 9
0 3 0
1
2 9
2 3
3 3
3 27
Make the Graph
• Now that you have a data table of ordered pairs for the function, you can plot
the points on a graph. (-2, 1/9) (0,1) (2,9)
Draw in the curve that fits the plotted points. y
y

x
x
Domain and Range
Domain – The collection of all input values of a function. These are usually the “x” values.
Range – The collection of all output values of a function. These are usually the “y” values.

Describe the domain and range of the function y = -5x.

Domain – The domain of the function is all real numbers since the function is
defined for all x-values.
Range – The range of the function is all negative real numbers.
Determine the intercepts, zeroes,
and asymptotes of an exponential
function
www.gcisd-k12.org/cms/.../unit%205%20Day%201%20Exponential%20Functions.ppt
Definition of Exponential Function

The exponential function f with base a is defined by

f(x) = ax

where a > 0, a 1, and x is any real number.

For instance,

f(x) = 3x and g(x) = 0.5x

are exponential functions.


Example: Exponential Function

The value of f(x) = 3x when x = 2 is


f(2) = 32 = 9
The value of f(x) = 3x when x = –2 is
1
f(–2) = 3–2 =
9

The value of g(x) = 0.5x when x = 4 is


g(4) = 0.54 = 0.0625
Graph of Exponential Function (a > 1)
The graph of f(x) = ax, a > 1 y

Exponential
Growth 4
Range: (0, )
Function

(0, 1)
x
4
Horizontal Asymptote
y=0
Domain: (– , )
Graph of Exponential Function (0 < a < 1)
The graph of f(x) = ax, 0 < a < 1 y

Exponential 4
Decay Range: (0, )
Function
(0, 1)

x
4
Horizontal Asymptote
y=0
Domain: (– , )
Exponential Function

3 Key Parts
1. Pivot Point (Common Point)
2. Horizontal Asymptote
3. Growth or Decay
Manual Graphing

• Lets graph the following together:

• f(x) = 2x
Example: Graph f(x) = 2x
Example: Sketch the graph of f(x) = 2x.

x f(x) (x, f(x)) y

-2 ¼ (-2, ¼)
4
-1 ½ (-1, ½)
2
0 1 (0, 1)
1 2 (1, 2) x
2 4 (2, 4) –2 2
Definition of the Exponential Function

The exponential function f with base b is defined by


f (x) = bx or y = bx
Where b is a positive constant other than and x is any real number.

Here are some examples of exponential functions.


f (x) = 2x g(x) = 10x h(x) = 3x

Base is 2. Base is 10. Base is 3.


Transformations Involving Exponential Functions
Transformation Equation Description

Horizontal g(x) = bx+c • Shifts the graph of f (x) = bx to the left c units if c > 0.
translation • Shifts the graph of f (x) = bx to the right c units if c < 0.

Vertical stretching or g(x) = cbx Multiplying y-coordintates of f (x) = bx by c,


shrinking • Stretches the graph of f (x) = bx if c > 1.
• Shrinks the graph of f (x) = bx if 0 < c < 1.

Reflecting g(x) = -bx • Reflects the graph of f (x) = bx about the x-axis.
g(x) = b-x • Reflects the graph of f (x) = bx about the y-axis.

Vertical translation g(x) = bx+ c • Shifts the graph of f (x) = bx upward c units if c > 0.
• Shifts the graph of f (x) = bx downward c units if c < 0.
Example: Translation of Graph
Example: Sketch the graph of g(x) = 2x – 1. State the domain and range.
The graph of this y f(x) = 2x
function is a vertical
translation of the 4
graph of f(x) = 2x
down one unit . 2

Domain: (– , )
Range: (–1, ) x
y = –1
Example: Reflection of Graph
• Example: Sketch the graph of g(x) = 2-x. State the domain and range.
The graph of this
function is a y f(x) = 2x
reflection the graph
of f(x) = 2x in the y-
4
axis.

Domain: (– , )
Range: (0, ) x
–2 2
Key Concepts of Inverse Functions,
Exponential Functions and
Logarithmic Functions - Part 004
Objectives

At the end of this module, the learner should be able to:


1. Graph exponential functions
2. Solve problems involving exponential functions, equations, and inequalities
3. Represent real-life situations using logarithmic functions
4. Distinguish logarithmic function, logarithmic equation, and logarithmic
inequality
Graph exponential functions
iws.collin.edu/jmerrill/1314/1314%20ppt.../4.1%20Exponential%20Functions.ppt
Definition of Exponential Functions

• The exponential function f with a base b is defined by f(x) = bx where b is a positive


constant other than 1 (b > 0, and b ≠ 1) and x is any real number.
• So, f(x) = 2x, looks like:
Graphing Exponential Functions
x
1
y y 7x
7

• Four exponential functions


have been graphed.
x
Compare the graphs of 1
y y 2x
functions where b > 1 to 2
those where b < 1
Graphing Exponential Functions

• So, when b > 1, f(x) has a graph that goes


up to the right and is an increasing
function.
• When 0 < b < 1, f(x) has a graph that goes
down to the right and is a decreasing
function.
Characteristics

• The domain of f(x) = bx consists of all real numbers (- , ). The


range of f(x) = bx consists of all positive real numbers (0, ).
• The graphs of all exponential functions pass through the point (0,1).
This is because f(o) = b0 = 1 (b o).
• The graph of f(x) = bx approaches but does not cross the x-axis. The
x-axis is a horizontal asymptote.
• f(x) = bx is one-to-one and has an inverse that is a function.
Transformations
y 2x

y 2x 3
• Vertical translation f(x) = bx + c
• Shifts the graph up if c > 0
• Shifts the graph down if c < 0 y 2x 4
Transformations
y 2x
• Horizontal translation:
g(x)=bx+c
y 2( x 3)

• Shifts the graph to the left if c


>0
y 2( x 4)

• Shifts the graph to the right if


c<0
Transformations

y 2x
• Reflecting
• g(x) = -bx reflects the graph x
about the x-axis. y 2

• g(x) = b-x reflects the graph


about the y-axis.
y 2x
Transformations
y 2x

• Vertical stretching or
shrinking, f(x)=cbx:
y 4(2 x )
• Stretches the graph if c > 1 y
1 x
(2 )
• Shrinks the graph if 0 < c < 1 4
Transformations

y 2x
• Horizontal stretching or shrinking,
f(x)=bcx: y 4(2 x )

• Shinks the graph if c > 1


• Stretches the graph if 0 < c < 1 y
1 x
4
(2 )
Practice

• Graph the function f(x) =


2(x-3) +2
• Where is the horizontal y=2
asymptote?
Practice

• Graph the function f(x) =


4(x+5) - 3
• Where is the horizontal y=-3
asymptote?
Solve problems involving exponential
functions, equations, and inequalities
teachers.henrico.k12.va.us/math/ito_08/05Exponential/5LES3/exp_log_eqns_n.ppt
Exponential Equations with Like Bases

• In an Exponential Equation, the variable is in the exponent. There may


be one exponential term or more than one, like…

or
• If you can isolate terms so that the equation can be written as two
expressions with the same base, as in the equations above, then the
solution is simple.
Exponential Equations with Like Bases

• Example #1 - One exponential expression.

1. Isolate the exponential expression


and rewrite the constant in terms of
the same base.

2. Set the exponents equal to each


other (drop the bases) and solve the
resulting equation.
Exponential Equations with Like Bases

• Example #2 - Two exponential expressions.


1. Isolate the exponential
expressions on either side of the =.
We then rewrite the 2nd expression
in terms of the same base as the first.

2. Set the exponents equal to each


other (drop the bases) and solve the
resulting equation.
Exponential Equations with Different Bases

• The Exponential Equations below contain exponential expressions


whose bases cannot be rewritten as the same rational number.

or

• The solutions are irrational numbers, we will need to use a log function
to evaluate them.
Exponential Equations with Different Bases

• Example #1 - One exponential expression.


1. Isolate the exponential expression.

2. Take the log (log or ln) of both


sides of the equation.

3. Use the log rule that lets you


rewrite the exponent as a multiplier.
Exponential Equations with Different Bases

• Example #1 - One exponential expression.


4. Isolate the variable.
Exponential Equations with Different Bases

• Example #2 - Two exponential expressions.


1. The exponential expressions are
already isolated.
2. Take the log (log or ln) of both
sides of the equation.
3. Use the log rule that lets you
rewrite the exponent as a multiplier
on each side..
Exponential Equations with Different Bases

• Example #2 - Two exponential expressions.


4. To isolate the variable, we
need to combine the ‘x’
terms, then factor out the ‘x’
and divide.
Represent real-life situations
using logarithmic functions
http://iws.collin.edu/jmerrill/1314/1314%20ppt_files/4.5%20Exp%20Growth%20and%20Decay.pp
t
Review
log
log x log x 66 33
log33xx log
3
log33 xx
3
6 3
log
log33xx xx 66 33
33 x x 6
27 x2 6x
x 2 6 x 27 0
x 9 x 3 0

Is this okay? x 9, 3
Arguments must be positive x 9
Review

500e0.3x = 600
e0.3x = 1.2
ln 1.2 = 0.3x
ln1.2
x
0.3

x = 0.608
Exponents and Logarithms

• How are exponents and logarithms related?


• They are inverses of each other
• Why is this important?
• Using inverses allow us to solve problems (we use subtraction to solve addition
problems & division to solve multiplication)
• Many real-life scenarios are exponential in nature and logarithms allow us to solve
for the unknown.
Examples Using Logarithmic Scales

• The Richter scale is used to determine the intensity of an earthquake.


• Measuring acidity using the pH scale, or concentration of ions.
• Carbon dating.
• Modeling population growth/decay--just to name a few…
Exponential Decay Model

• A(t) = A0e kt
• A0 is the initial amount
• K is the growing/decay entity. If k>0, the entity is growing (an increasing function).
If k<0, the entity is decaying (a decreasing function).
• Looks like A(t) = Pert? It works the same way.
Population Model

• In 1970, the US population was 203.3 million. In 2003, the population was 294
million.

1. Find the exponential growth model


2. By which year will the US population reach 315 million?
Population Model
• t is the number of years after 1970.
• t=0 represents 1970. t = 33 represents 2003
• When t = 33, A = 294

• A(t) = A0ekt
• 294 = 203.3ek(33)
Population Con’t
What do you do
when the exponent
is a variable?

294
e33k
203.3 294

• 294 = ln 203.3 k 0.011


294
ln ln e33k
203.3ek(33) 203.3
33
294
ln 33kln e So, k ≈ 0.011, which is
203.3 exponential growth

What does The growth model is


lne = ?
A(t) = 203.3e0.011t
Population Con’t

• When will the population reach 315 million?


• A(t) = 203.3e0.011t
• 315 = 203.3e0.011t
• You finish…

• Did you get approximately 40?


• That means that in the year 2010 the population will be approx. 315 million!
Carbon Dating

• The natural base, e, is used to


estimate the ages of artifacts.
Plants and animals absorb Carbon-
14 from the atmosphere. When a
plant or animal dies, the amount
of carbon-14 it contains decays in
such a way that exactly half of the
initial amount is present after
5,715 years.
Carbon Dating

• The function that models the


decay of carbon-14, where A0
is the initial amount of
carbon-14, and A(t) is the
amount present t years after
the plant or animal dies, is

A(t ) A0e kt
Carbon Dating Example

• Archaeologists find scrolls and claim that they are 2000 years old. Tests indicate
that the scrolls contain 78% of their original carbon-14. Could the scrolls be 2000
years old?
• Using the same process as the last example, we find k to be -0.00012.
• Finding k is written out in the book on P449.
Carbon Dating Example

78% of the original

A(t ) A0e 0.00012t amount

.78A0 A0e 0.00012t

.78 e 0.00012t

ln .78 .00012t ln e
2070.5 t
Practice

• A wooden chest is found and said to be from the 2nd century BCE. Tests on a
sample of wood from the chest reveal that it contains 92% of its original carbon-
14. Could the chest be from the 2nd century BCE?
• Use the same k as the last example.
Practice

A(t ) A0e 0.00012t

.92A0 A0e 0.00012t

.92 e 0.00012t

ln .92 .00012t ln e
694.85 t
Logistic Growth Model

• The spread of disease is exponential in nature. However, there aren’t an infinite


number of people. Eventually, the disease has to level off. Growth is always
limited. A logistic growth model is used in this type of situation:

c
f(t)
1 ae bt

• Y = c is the horizontal asymptote. Thus c is the limiting value of the function.


Modeling the Spread of the Flu

• The function below describes the number of people, f(t), who have become ill with
influenza t weeks after its initial outbreak in a town with a population of 30,000
people.

30, 000
f(t)
1 20e 1.5t
Modeling the Spread of the Flu

1. How many people became ill with the flu when the epidemic began?
2. How many people were ill by the end of the fourth week?
3. What is the limiting size of f(t), the population that become ill?
30, 000
f(t)
1 20e 1.5t
Modeling the Spread of the Flu
1. How many people became ill with the flu when the epidemic began?
• In the beginning, t = 0:

30, 000
f(t)
1 20e 1.5(0)

30, 000
f(t) 1429
1 20
Modeling the Spread of the Flu
2. How many people were ill by the end of the fourth week?

30, 000
f(t)
1 20e 1.5(4)

f(t) 28,583
Modeling the Spread of the Flu
3. What is the limiting size of f(t), the population that become ill?

C represents the limiting size that f(t)


can obtain. There are only 30,000
people in the town, therefore, the
limiting size must be 30,000!
Distinguish logarithmic function,
logarithmic equation, and
logarithmic inequality
www.sd27j.org/cms/lib8/CO01900701/Centricity/Domain/568/4.5%20A2.ppt
An exponential equation is an equation containing
one or more expressions that have a variable as an
exponent. To solve exponential equations:

• Try writing them so that the


bases are all the same.
• Take the logarithm of both
sides.
Helpful Hint
When you use a rounded number in a check, the
result will not be exact, but it should be
reasonable.
Example 1A: Solving Exponential Equations
Solve and check.

98 – x = 27x – 3 Rewrite each side with the same


base; 9 and 27 are powers of 3.
(32)8 – x = (33)x – 3

316 – 2x = 33x – 9 To raise a power to a power,


multiply exponents.

16 – 2x = 3x – 9 Bases are the same, so the


exponents must be equal.
x=5 Solve for x.
Example 1B: Solving Exponential Equations
Solve and check.
4x – 1 = 5
log 4x – 1 = log 5 5 is not a power of 4, so take the
log of both sides.
(x – 1)log 4 = log 5 Apply the Power Property of
Logarithms.
log5
x –1 = log4 Divide both sides by log 4.
log5
x = 1 + log4 ≈ 2.161

Check Use a calculator.


The solution is x ≈ 2.161.
Check It Out! Example 1a
Solve and check.
32x = 27 Rewrite each side with the same
(3)2x = (3)3 base; 3 and 27 are powers of 3.

32x = 33 To raise a power to a power,


multiply exponents.

2x = 3 Bases are the same, so the


exponents must be equal.
x = 1.5 Solve for x.
Check It Out! Example 1b
Solve and check.
7–x = 21
21 is not a power of 7, so take the
log 7–x = log 21
log of both sides.

(–x)log 7 = log 21 Apply the Power Property of


Logarithms.
log21
–x = log7 Divide both sides by log 7.

log21
x = – log7 ≈ –1.565
Check It Out! Example 1c
Solve and check.
23x = 15
log23x = log15 15 is not a power of 2, so take the
log of both sides.

(3x)log 2 = log15 Apply the Power Property of


Logarithms.
log15 Divide both sides by log 2,
3x = log2
then divide both sides by 3.

x ≈ 1.302
Example 3A: Solving Logarithmic Equations

Solve.
log6(2x – 1) = –1
log (2x –1)
6 6 = 6–1 Use 6 as the base for both sides.

2x – 1 = 1 Use inverse properties to remove


6
6 to the log base 6.
7
x = 12 Simplify.
Example 3B: Solving Logarithmic Equations

Solve.
log4100 – log4(x + 1) = 1

log4( x + 1 ) = 1
100 Write as a quotient.
100
log4( x + 1 ) Use 4 as the base for both sides.
4 = 41
100
x+1
=4 Use inverse properties on the
left side.
x = 24
Example 3C: Solving Logarithmic Equations

Solve.
log5x 4 = 8

4log5x = 8 Power Property of Logarithms.

log5x = 2 Divide both sides by 4 to isolate log5x.

x = 52 Definition of a logarithm.

x = 25
Example 3D: Solving Logarithmic Equations

Solve.

log12x + log12(x + 1) = 1

log12 x(x + 1) = 1 Product Property of Logarithms.

12 log 12
x(x +1)
= 121 Exponential form.

x(x + 1) = 12 Use the inverse properties.


Example 3 Continued
x2 + x – 12 = 0 Multiply and collect terms.
(x – 3)(x + 4) = 0 Factor.
x – 3 = 0 or x + 4 = 0 Set each of the factors equal to zero.
x = 3 or x = –4 Solve.
Check Check both solutions in the original equation.
log12x + log12(x +1) = 1 log12x + log12(x +1) = 1
log123 + log12(3 + 1) 1 log12( –4) + log12(–4 +1) 1 x
log123 + log124 1
log12( –4) is undefined.
log1212 1
1 1 The solution is x = 3.
Check It Out! Example 3a
Solve.
3 = log 8 + 3log x

3 = log 8 + 3log x
3 = log 8 + log x3
Power Property of Logarithms.
3 = log (8x3) Product Property of Logarithms.
3)
103 = 10log (8x Use 10 as the base for both sides.
1000 = 8x3 Use inverse properties on the
right side.
125 = x3
5=x
Check It Out! Example 3b
Solve.

2log x – log 4 = 0

2log( 4 )=0
x Write as a quotient.
x

2(10 log 4
) = 100 Use 10 as the base for both sides.

2( x ) = 1 Use inverse properties on the


4
left side.
x=2
Example 4A: Using Tables and Graphs to Solve
Exponential and Logarithmic Equations and Inequalities
Use a table and graph to solve 2x + 1 > 8192x.
Use a graphing calculator. Enter 2^(x + 1) as Y1
and 8192x as Y2.

In the table, find the x-values In the graph, find the x-value
where Y1 is greater than Y2. at the point of intersection.
The solution set is {x | x > 16}.
Key Concepts of Inverse Functions,
Exponential Functions and
Logarithmic Functions - Part 005
Objectives

At the end of this module, the learner should be able to:


1. Illustrate the laws of logarithms.
2. Solve logarithmic equations and inequalities
3. Represent a logarithmic function through its: (a) table of values, (b) graph, and (c)
equation.
4. Find the domain and range of a logarithmic function.
Illustrate the laws of logarithms.
www.elcamino.edu/faculty/rho/M130%20Fall%202013/chapter4_Ed6_4_4.ppt
Laws of Logarithms

• Since logarithms are exponents,


the Laws of Exponents give rise to
the Laws of Logarithms.
Laws of Logarithms

• Let a be a positive number, with a ≠ 1.

• Let A, B, and C be any real numbers with A > 0 and B >


0.
Law 1

loga ( AB ) loga A loga B

• The logarithm of a product of numbers


is the sum of the logarithms of the numbers.
Law 2
A
loga loga A loga B
B
• The logarithm of a quotient of numbers
is the difference of the logarithms of
the numbers.
Law 3
C
loga A C loga A
• The logarithm of a power of a number
is the exponent times the logarithm of
the number.
Laws of Logarithms—Proof

• We make use of the property


loga ax = x
Law 1—Proof
•Let loga A = u and loga B = v.
• When written in exponential form, au A and av B
these equations become:
• Thus,
loga ( AB ) loga (au av ) loga (au v )
u v
loga A loga B
Law 2—Proof

• Using Law 1, we have:


A A
loga A loga B loga loga B
B B
A
• Thus, loga
B
loga A loga B
Law 3—Proof

• Let loga A = u.

• Then, au = A.
C
C u uC
loga A loga a loga a
• So,
uC C loga A
E.g. 1—Using the Laws to Evaluate Expressions

(a) log4 2 log4 32

• Evaluate each expression.


(b) log2 80 log2 5

(c) 1
3
log8
E.g. 1—Evaluating Expressions
Example (a) log4 2 log4 32

log4 (2 32) Law 1

log4 64

3 Because 64 = 4
3
E.g. 1—Evaluating Expressions
log2 80 log2 5
Example (b)

80
log2 Law 2
5

log2 16

4 Because16 = 2
4
E.g. 1—Evaluating Expressions
Example (c)
1
3
log8

1/ 3
log8 Law 3

log 1
2
Property of negative exponents

0.301 Calculator
Solve logarithmic equations and
inequalities
eachers.henrico.k12.va.us/math/ito_08/05Exponential/5LES3/exp_log_eqns_n.ppt
Logarithmic Equations

• In a Logarithmic Equation, the variable can be inside the log function


or inside the base of the log. There may be one log term or more than
one. For example …
Logarithmic Equations

• Example 1 - Variable inside the log function.

1. Isolate the log expression.

2. Rewrite the log equation as an


exponential equation and solve
for ‘x’.
Logarithmic Equations

• Example 2 - Variable inside the log function, two log expressions.


1. To isolate the log expression, we 1st
must use the log property to combine a
difference of logs.

2. Rewrite the log equation as an


exponential equation (here, the base is
‘e’).
3. To solve for ‘x’ we must distribute
the ‘e’ and then collect the ‘x’ terms
together and factor out the ‘x’ and
divide.
Logarithmic Equations

• Example 3 - Variable inside the base of the log.


1. Rewrite the log equation as an
exponential equation.

2. Solve the exponential equation.


Represent a logarithmic function
through its: (a) table of values, (b)
graph, and (c) equation
www.saigontech.edu.vn/faculty/TrungLM/ppt/lca10_0403.ppt
Logarithmic Function
Exponential and
logarithmic functions
are inverses of each
other. The graph of
y = 2x is shown in red.
The graph of its inverse
is found by reflecting
the graph across the
line y = x.
Logarithmic Function

The graph of the


inverse function,
defined by y = log2 x,
shown in blue, has the
y-axis as a vertical
asymptote.
Logarithmic Function

• Since the domain of an exponential function is the set of all real numbers, the
range of a logarithmic function also will be the set of all real numbers. In the
• same way, both the range of an exponential function and the domain of a
logarithmic function are the set of all positive real numbers, so logarithms can be
found for positive numbers only.
For (x) = log2 x: f ( x ) loga x
x (x)
¼ –2
½ –1
1 0
2 1
4 2 (x) = loga x, a > 1, is
8 3 increasing and continuous on
its entire domain, (0, ) .
For (x) = log2 x: f ( x ) loga x
x (x)
¼ –2
½ –1
1 0
2 1
4 2 The y-axis is a vertical asymptote
8 3 as x 0 from the right.
For (x) = log2 x: f ( x ) loga x
x (x)
¼ –2
½ –1
1 0
2 1
4 2
8 3 The graph passes through the points
1
, 1 , 1,0 , and a,1 .
a
For (x) = log1/2 x: f ( x ) loga x
x (x)
¼ 2
½ 1
1 0
2 –1
4 –2 (x) = loga x, 0 < a < 1, is
8 –3 decreasing and continuous on its
entire domain, (0, ) .
For (x) = log1/2 x: f ( x ) loga x
x (x)
¼ 2
½ 1
1 0
2 –1
4 –2 The y-axis is a vertical
8 –3 asymptote as x 0 from the
right.
For (x) = log1/2 x: f ( x ) loga x
x (x)
¼ 2
½ 1
1 0
2 –1
4 –2
The graph passes through
8 –3 the points 1 , 1 , 1,0 , and a,1 .
a
Find the domain and range of a
logarithmic function
www.saigontech.edu.vn/faculty/TrungLM/ppt/lca10_0403.ppt
Caution

• If you write a logarithmic function in exponential form, choosing y-values to


calculate x-values, be careful to write the values in the ordered pairs in the correct
order.
Example 3
• Graph each function. Give the domain and range.
• a. f ( x ) log2 ( x 1)
Solution The graph of (x) = log (x – 1) is the graph of
2
(x) = log2 x translated 1 unit to the right. The vertical
asymptote is x = 1. The domain of this function is (1, ) since
logarithms can be found only for positive numbers. To find some
ordered pairs to plot, use the equivalent exponential form of the
equation
y = log2 (x – 1).
Example 3
•Graph each function. Give the domain and range.
a. f ( x ) log2 ( x 1)
Solution y log2 ( x 1)
y Write in exponential
x 1 2 form.
y
x 2 1 Add 1.
Example 3
•Graph each function. Give the domain and range.
a. f ( x ) log2 ( x 1)
Solution
We choose values for y
and then calculate each
of the corresponding x-
values. The range is
(– , ).
Example 3
•Graph each function. Give the domain and range.
b. f ( x ) (log3 x ) 1
Solution
The function defined by (x) = (log3 x) – 1 has the
same graph as g(x) = log3 x translated 1 unit down.
We find ordered pairs to plot by writing y = (log3 x) – 1
in exponential form.
Example 3
• Graph each function. Give the domain and range.
b. f ( x ) (log3 x ) 1
Solution y (log3 x ) 1

y 1 log3 x Add 1.

y 1
x 3 Write in exponential form.
Example 3
•Graph each function. Give the domain and range.
b. f ( x ) (log3 x) 1
Solution
Again, choose y-
values and calculate
the corresponding x-
values. The domain is
(0, ) and the range is
(– , ).
Key Concepts of Inverse Functions,
Exponential Functions and
Logarithmic Functions - Part 006
Objectives

At the end of this module, the learner should be able to:


1. Determine the intercepts, zeroes, and asymptotes of logarithmic functions
2. Graph logarithmic functions
3. Solve problems involving logarithmic functions, equations, and inequalities
Determine the intercepts, zeroes,
and asymptotes of logarithmic
functions
www.mathxtc.com/Downloads/NumberAlg/files/Logarithmic%20Functions.ppt
Ex: Find the domain, asymptote(s), and intercept(s) of
each function.

• To find domain of logs, set whatever is in the log to be greater than zero!
• The vertical asymptote will be the same as the value in the domain!
• Find the intercepts by setting one of the variables equal to zero, then
solving!
Ex: Find the domain, asymptote(s), and intercept(s) of
each function.
• f(x) = log3(2x)
• Domain: 2x > 0, so x > 0
• Asymptote: Since domain is x > 0, then asymptote is at x = 0
• Intercepts:
•Y-int: none; can’t have zero in the log
•Also, y-axis is a vertical asymptote
•X-int: 0 = log3(2x) 30 = 2x 1 = 2x …
•(½ , 0)
Ex: Find the domain, asymptote(s), and intercept(s) of
each function.
• f(x) = log(x + 2) + 1
• Domain: x + 2 > 0, so x > -2
• Asymptote: x = -2
• Intercepts:
•Y-int: occurs when x = 0, so log(2) + 1 (0, 1.301)
•X-int: occurs when y = 0, so…
•-1 = log(x+2) 10-1 = x+2 .1 = x+2 …
•(-1.9, 0)
Graph logarithmic functions
http://www.grantbulldogs.org/site/handlers/filedownload.ashx?moduleinstanceid=1714&dataid=
1314&FileName=3.2%20-%20Logarithmic%20Functions%20and%20Their%20Graphs.ppt
Graph f (x) = log2 x
Since the logarithm function is the inverse of the exponential function of the same
base, its graph is the reflection of the exponential function in the line y = x.
y y = 2x y=x
x 2x
–2 1 horizontal y = log2 x
4 asymptote y = 0

–1 1 x-intercept
2 x
0 1 (1, 0)

1 2
vertical asymptote
2 4 x=0
3 8
Graph the common logarithm function f(x) = log10 x

x 1 1 1 2 4 10
100 10 by calculator
f(x) = log10 x –2 –1 0 0.301 0.602 1
y
f(x) = log10 x

x
5
(0, 1) x-intercept

x=0
vertical
asymptote –5
The graphs of logarithmic functions are similar for different
values of a. f(x) = loga x (a 1)
y-axis y
Graph of f (x) = loga x (a 1) y=ax y=x
vertical
1. domain (0, ) asymptote
y = log2 x
2. range ( , )
3. x-intercept (1, 0)
4. vertical asymptote domain
x
x 0 as x 0 f ( x) x-intercept
5. increasing (1, 0)
6. continuous
7. one-to-one range
8. reflection of y = a x in y = x
y y = ln x
The function defined by
f(x) = loge x = ln x
x
(x 0, e 2.718281 ) 5

is called the natural


logarithm function.
–5

y = ln x is equivalent to e y = x

Use a calculator to evaluate: ln 3, ln –2, ln 100


Function Value Keystrokes Display
ln 3 LN 3 ENTER 1.0986122
ln –2 LN –2 ENTER ERROR
ln 100 LN 100 ENTER 4.6051701
Copyright © by Houghton Mifflin Company, Inc. All rights reserved. 11
Solve problems involving logarithmic
functions, equations, and inequalities
sylmarhs.org/ourpages/auto/2012/8/19/40866437/A2CH7L5.ppt
Example 3A: Solving Logarithmic Equations

Solve.

log6(2x – 1) = –1

6 log6 (2x –1) = 6–1 Use 6 as the base for both sides.

2x – 1 = 1 Use inverse properties to remove 6 to the


6 log base 6.

x= 7 Simplify.
12
Example 3B: Solving Logarithmic Equations

Solve.

log4100 – log4(x + 1) = 1

log4 ( 100
x+1 =1 ) Write as a quotient.

log4 ( x100
+)1 = 4
1 Use 4 as the base for both sides.
4

100 = 4
x+1 Use inverse properties on the left side.

x = 24
Example 3C: Solving Logarithmic Equations

Solve.

log5x 4 = 8

4log5x = 8 Power Property of Logarithms.


log5x = 2

x = 52 Divide both sides by 4 to isolate log5x.

x = 25 Definition of a logarithm.
Example 3D: Solving Logarithmic Equations

Solve.

log12x + log12(x + 1) = 1

log12 x(x + 1) = 1 Product Property of Logarithms.

12 +1)121
log12x(x = Exponential form.

x(x + 1) = 12 Use the inverse properties.


Example 3 Continued
x2 + x – 12 = 0 Multiply and collect terms.
(x – 3)(x + 4) = 0 Factor.

x – 3 = 0 or x + 4 = 0 Set each of the factors equal to zero.

x = 3 or x = –4 Solve.

Check Check both solutions in the original equation.

log12x + log12(x +1) = 1 log12x + log12(x +1) = 1

log123 + log12(3 + 1) 1 log12( –4) + log12(–4 +1) 1 x


log123 + log124 1
log12( –4) is undefined.
log1212 1
1 1 The solution is x = 3.
Check It Out! Example 3a
Solve.

3 = log 8 + 3log x

3 = log 8 + 3log x

3 = log 8 + log x3
Power Property of Logarithms.
3 = log (8x3)
Product Property of Logarithms.
103 = 10log (8x3)
Use 10 as the base for both sides.
1000 = 8x3
Use inverse properties on the right side.
125 = x3

5=x
Check It Out! Example 3b
Solve.

2log x – log 4 = 0

2log ( x4) = 0 Write as a quotient.

x
2(10log ) 4= 100 Use 10 as the base for both sides.

2( x ) = 1
4 Use inverse properties on the left side.

x=2
WEEK11
Simple Interest and Simple Discount

When people need to secure funds for some purposes, one of the ways they
usually resort to is borrowing. On the other hand, the person or institution, which lends the
money would also wish to get something in return for the use of the money. The person
who borrows money for any purpose is a debtor or maker and the person or institution,
which loans the money, is the lender

The payment for the use of borrowed money is called interest. The capital or sum
of the money invested is called the principal. The fractional part of the principal that is
paid on the loan is the rate of interest and is usually expressed as percent. The time or
term of the loan is the number of units (days, months, and years) of the time for which the
money is borrowed and for which interest is calculated. The sum of the principal and the
interest which is accumulated at a certain time, is referred to as Final Amount or Maturity
Value. The amount received by the borrower is the Present Value or Proceeds of the loan.

Two types of Interest

Simple Interest is interest in which only the original principal bears interest for the
entire term of the loan. Here the principal and present value is equal.

Compound interest is interest added to the principal at the end of a certain period
of time after which the interest is computed on the new principal, and this process is
repeated until at the end of the loan is reached.

To elucidate the above concept, suppose Mr. Ariel borrows P10, 000 at the rate of
12% per year. Here P10, 000 is the principal, 12% is the rate of interest and the time is one
year. If the loan is a simple interest loan, then the interest on P10,000 is P1, 200 At the end
of one year Mr. Ariel should pay the lender a total amount of P11,200 which is called the
maturity value.

1.1 Simple Interest

It is define as the product of principal, rate, and time.

I P r t

Where;
I = simple interest in pesos
P = principal in pesos
r = rate or percent of the principal which is to be paid per unit of time
t = unit of time usually expressed in years of or fractional part of year

Other formula can be derived from the above.

I
Principal ( P ) P
r t
I
Rate ( r ) r
P t
I
Time ( t ) t
P r

Final Amount Formula


F=P+I
and Interest
I=F-P

Ordinarily, in business, the interest rate is expressed as percent. In computing the


interest it is necessary to convert the rate into a fraction or decimal equivalent such
10%=10/100 or 0.01; 4 ½% = 4.5% = 4.5/100 = 0.045.
The term of the loan is the period during which the borrower has the use of all or
part of the borrowed money. The term or time may be stated in any of the following ways.

1. When the expressed in number of year(s). our formula will be


I P r number of year (s)
2. When the time is expressed in number of month(s)

number of month(s)
I P r
12
3. When the time is expressed in number of days, there are two (2) ways of \
computing interest.
a. Ordinary interest (Io)
number of days
I P r
360
b. Exact interest (Ie)
number of days
I P r
365
4. When the time is expressed between dates, there are four (4) ways of
computing interest.
a. Io – Actual time
actual number of days
I P r
360
b. Io – Approximate time
approximate number of days
I P r
360
c. Ie – Actual time
actual number of days
I P r
365
d. Ie – Approximate time
approximate number of days
I P r
365
Let us work on the following examples to illustrate the application of the formula.
Illustrative Examples
1. Find the interest and amount on P1000 at 7½% simple interest for 5 years.

Solution:

Interest Final Amount


I=Pxrxt F=P+I
= 1000 x 7½% x 5 = 1000 + 375
= 1000 x ( 7.5/100) x 5 = P1,375
= 1000 x 0.075 x 5
= P 375
2. Find the interest and amount on P900 at 7¼% simple interest for 9 months.

Solution:

Interest Final Amount


number of months
I P r F=P+I
12
= 900 x 7¼% x (9/12) = 900 + 48.94
= 900 x ( 7.25/100) x 0.75 = P948.94
= 900 x 0.0725 x 0.75
= P48.94
3
3. Find the interest and amount on P1200 at 11 % simple interest for 2 years and 5 months.
8
Solution:
Interest Final Amount
number of months
I P r years F=P+I
12
= 1200 x 0.11375 x 2 + (5/12) = 1200 + 329.88
= 1200 x 0.11375 x (29/12) = P1,529.88
= 1200 x 0.11375 x (29/12)
= P329.88
4. If a principal of P2500 earns interest of P185 in 3 years and 3 months, what interest rate,
is in effect?
Solution:

1
r 100%
P t
1
r 100%
2500 3.25

r 2.28%

5. A principal earns interest of P385 in 2 years and 9 months at simple interest rate of 9½%.
Find the principal invested.
Solution:

1
P
r t
385
P
0.95 2.75

P P1, 473.68

6. How long will it take for P8,000 to earn P2400, if it is invested at 6½% simple interest?
Solution:

I
t
P r
2400
t
8000 0.065

t 4.62 years
4.62 years = 4 years
.62 x 12 mos = 7.44 or t months
.44 x 30 days = 13.2 or 13 days
.2 x 24 hrs = 4.8 or 4 hours
.8 x 60 minutes = 48.0 or 48 minutes
7. Mr. Villoria borrowed P15,000 from a bank at 11¼% simple interest for 2 years and 9
months. How much did Mr. Villoria pay back the bank.

Given: P = P15,000 r = 11¼% = .1125 t = 2 9/12 or 2.75


Solution:
I=Pxrxt F=P+I
= 15,000 x .1125 x 2.75 = 15,000 + 4,640.63
= P4,640.62 = P19,640.62

Exercises.

1. Mr. Mendoza borrowed P45,000 for 2 years and 8 months and paid P3,750 interest.
What was the simple interest rate? Ans. 3.125%

2. Ms. Santos loans P18,500 at 9½% simple interest, how long will it take her to get P6,800
interest? Ans. 3.87 years

3. Four months after borrowing money, Ms. Lundag pays an interest of P2,750. How much
did she borrow if the simple interest rate is 12 1/8%? Ans. P68041.24

4. In what time will P4,850 double itself at the rate of 9¼% simple interest? Ans. 10.81years

5. If a principal of P3,000 earns interest of P500 in 3 years and 9 months, what interest rate,
is in effect? Ans. 4.444%
Amount and Present Value at Simple Interest

At the end of the term for which interest is to be computed, the amount due on a lender
is the sum of the principal and interest and is designed by the symbol (F). This is called the
Final Amount or Maturity Value and is given by the formula

F=P+I
F = P + Prt
By factoring we have F = P(1 + rt ).

A. Accumulation is the process of determining the amount F of a given principal P due


at a specified time t. To accumulate a principal P for t years means to solve for the final
amount by applying the formula

F = P(I + rt )
B. Discounting is the process of determining the present value P of any amount due in the
future. To discount the amount F for t years, means to solve P applying the formula

F
P=
1+rt

Let us work on the following examples to illustrate the application of the above.

Illustrative examples:

1. A businessman borrows P9,900 for 2 years at 9¼% simple interest. What amount must
he repay?

Given: F = P9,900 r = 9¼% = 9.25% = 0.0925 t= 2 years

Solution:

F =P (1 + rt )

= 9,900 (1+(0.0925 x 2))


= 9,900(1+ 0.185)
= 9,900(1.185)
= P11,731.50
2. Accumulate P11,800 for 115 days at 11 1/8% simple interest

Given: P = P11,800 r = 11 1/8% = 0.11125 t = 115 days

Solution:

F =P (1 + rt )

= 11,800 (1+(0.11125 x (115/360))


= 11,800(1.035538)
= P12,219.35

3. Accumulate P10,100 for 8 months at 10 4/5% simple interest.


Given: P = P10,100 r = 10 4/5% = 0.108 t = 8 months
Solution:

F =P (1 + rt )

= 10,100 (1+(0.108 x (8/12))


= 10,100(1+0.072)
= 10,100(1.072)
= P10,827.20
4. Discount P5,500 for 11 months at 14¼% simple interest.
Given: F = P5,500 r = 14¼% = 0.1425 t = 11 months
Solution:

P = F / (1 + rt )

= 5,500 / (1+(0.1425 x (11/12))


= 5,500 / (1+0.130625)
= 5,500 / (1.130625)
= P4864.57
5. Discount P7,800 for 125 days at 7 7/8% simple interest
Given: F = P7,800 r = 7 7/8% = 0.07875 t=125 months

Solution:

P = F / (1 + rt )

= 7,800 / (1+(0.7875 x (125/360))


= 7,800 / (1+0.02734375)
= 7,800 / (1.02734375)
= P7,592.40
Week 13
Compound Interest

Simple interest as a type of interest has been discussed in the previous chapter.
The present discussion will focus on compound interest as another type.

Compound interest is the interest resulting from the periodic addition of simple
interest to the principal. When interest is periodically added to the principal and this new
sum is used as the new principal for a certain number of periods, the resulting value is
called Final or Compound Amount and designated by F.

The time between successive interest computations is called Compounding or


Conversion Period. The number of conversion periods for one year is denoted by m while
the total number of conversion periods for the whole investment term is denoted by n.
Conversion periods are usually expressed by any convenient length of time, and usually
taken as an exact division of the year, such as monthly, quarterly, semi – annually, and
annually,

When the conversion period are:

Annually m=1
Semi – Annually m=2
Quarterly m=4
Monthly m = 12
The total number of conversion periods for the whole term can be found from the
relation
n = time x number of conversion period per year (m)
n=txm
Thus, the term 5 years compounded:
Annually 5x1 n=5
Semi – Annually 5x2 n = 10
Quarterly 5x4 n = 20
Monthly 5 x 12 n = 60
The interest rate r is usually expressed as an annual or yearly rate, and must be
changed to the interest rate per unit conversion period or periodic rate I and can be
found from the relation;
interest rate r
i
conversion period per year m
r
i
m

Thus, the interest rate at 5% compounded;


Annually 5% 1 i = 5% = 0.05
1
Semi – Annually 5% 2 i = 2 % 0.025
2
1
Quarterly 5% 4 i = 1 % 0.0125
4
5
Monthly 5% 12 i= % 0.4166
12
The fundamental formula for compound amount is
F = P (1 + I )n
I=F–P
where;
F = final amount
P = original principal
i = periodic rate
n = total number of conversion period for the whole term ( n = t x m)
I = compound interest

Accumulation Factor Method


As mentioned in chapter 11 accumulation of the principal P is the process of
finding the final amount F, and the simplest method in finding the compound amount is
by the use of accumulation factor method.
Illustrative Example

1. Find the compound amount and interest if P5,500 is invested at 8% compounded


quarterly for 5 years and 6 months.

Given: P = P5,500 r = 8%, m = 4 t = 5yrs & 6mos.

Solution:

i = r/m n=txm
= 8% / 4 = 5 6/12 x 4
= 2% = 5.5 x 4
= 0.02 = 22

n
F=P 1 i
22
5, 500 1 0.02
22
5, 500 1.02
5,500 1.545980
P8,502.89

2. Accumulate P8,400 for 2 years at 7% converted monthly.


Given: P = P8,400 r = 7%, m = 12 t = 2 years

Solution:

i = r/m n=txm
= 7% / 12 = 2 x 12
= .005833 = 24

n
F=P 1 i
24
8, 400 1 0.005833
24
8, 400 1.005833
8,400 1.149806
P9,658.37
3. What sum of money will be required to settle a loan of P8,700 for 8 years and 6 months
at a rate of 9% compounded quarterly?

Given: P = P8,700 r = 9%, m = 4 t = 8 years and 6 months

Solution:

i = r/m n=txm
= 9% /4 = 8 6/12 x 4
= 2.25% = 8.5 x 4
=0.0225 = 34

n
F=P 1 i
34
8, 700 1 0.0225
34
5, 500 1.0225
5,500 2.130849
P18,538.39

4. Accumulate P15,400 for 5 years and 8 months at 5 ½% compounded semi – annually.

Given: P = P15,400 r = 5 ½%, m = 2 t = 5yrs and 8 mos.

Solution:

i = r/m n=txm
= 5 ½% /2 = (5 8/12) x 2
= 2.75% = (68 /12) x 2
=0.0275 = (17/3) x 2 = 34/3

n
F=P 1 i
34 / 3
15, 400 1 0.0275
34 / 3
15, 400 1.0275
15,400 1.359964
P20,943.44
5. If you deposit P5000 into an account paying 6% annual interest compounded monthly,
how long until there is P8000 in the account?

Given: F = P8,000 r = 6%, m = 12 P = P5,000

12 t
0.06
8000 5000 1
12
12 t
8000 5000 1 0.005
12 t
8000 5000 1.005
12 t
8000 5000 1.005
5000 5000
1.6 1.00512t

Take the logarithmic of both sides

log1.6 = log1.00512t
log1.6 = 12t log1.005
log1.6 12t log1.005
=
log1.005 log1.005
log1.6
= 12t
log1.005
94.23553232 12t
94.23553232 12t
12 12
t 7.9

6. Find the rate compounded quarterly if P745 accumulates to P786 in 3 years and 9
months.

Given: P = P745 F = 786 t = 3yrs and 9mos. m=4

Solution:
1
F n
r m 1 100%
p 1
4 1.055033557 15 1 100%
1
786 3.75 4
4 1.00357789 1 100%
4 1 100%
745
4 .00357789 100%
1
786 15 0.0143 100%
4 1 100%
745 1.43%

Practice Problems

Now it is your turn to try a few practice problems on your own.

Problem 1: If you deposit P4500 at 5% annual interest compounded quarterly, how muc
h money will be in the account after 10 years?

Problem 2: If you deposit P4000 into an account paying 9% annual interest compounde
d monthly, how long until there is P10000 in the account?

Problem3: If you deposit $2500 into an account paying 11% annual interest compounde
d quarterly, how long until there is P4500 in the account?

Problem4: How much money would you need to deposit today at 5% annual interest co
mpounded monthly to have P20000 in the account after 9 years?

Problem 5: If you deposit P6000 into an account paying 6.5% annual interest compound
ed quarterly, how long until there is P12600 in the account?
Week 13
Compound Interest

Simple interest as a type of interest has been discussed in the previous chapter.
The present discussion will focus on compound interest as another type.

Compound interest is the interest resulting from the periodic addition of simple
interest to the principal. When interest is periodically added to the principal and this new
sum is used as the new principal for a certain number of periods, the resulting value is
called Final or Compound Amount and designated by F.

The time between successive interest computations is called Compounding or


Conversion Period. The number of conversion periods for one year is denoted by m while
the total number of conversion periods for the whole investment term is denoted by n.
Conversion periods are usually expressed by any convenient length of time, and usually
taken as an exact division of the year, such as monthly, quarterly, semi – annually, and
annually,

When the conversion period are:

Annually m=1
Semi – Annually m=2
Quarterly m=4
Monthly m = 12
The total number of conversion periods for the whole term can be found from the
relation
n = time x number of conversion period per year (m)
n=txm
Thus, the term 5 years compounded:
Annually 5x1 n=5
Semi – Annually 5x2 n = 10
Quarterly 5x4 n = 20
Monthly 5 x 12 n = 60
The interest rate r is usually expressed as an annual or yearly rate, and must be
changed to the interest rate per unit conversion period or periodic rate I and can be
found from the relation;
interest rate r
i
conversion period per year m
r
i
m

Thus, the interest rate at 5% compounded;


Annually 5% 1 i = 5% = 0.05
1
Semi – Annually 5% 2 i = 2 % 0.025
2
1
Quarterly 5% 4 i = 1 % 0.0125
4
5
Monthly 5% 12 i= % 0.4166
12
The fundamental formula for compound amount is
F = P (1 + I )n
I=F–P
where;
F = final amount
P = original principal
i = periodic rate
n = total number of conversion period for the whole term ( n = t x m)
I = compound interest

Accumulation Factor Method


As mentioned in chapter 11 accumulation of the principal P is the process of
finding the final amount F, and the simplest method in finding the compound amount is
by the use of accumulation factor method.
Illustrative Example

1. Find the compound amount and interest if P5,500 is invested at 8% compounded


quarterly for 5 years and 6 months.

Given: P = P5,500 r = 8%, m = 4 t = 5yrs & 6mos.

Solution:

i = r/m n=txm
= 8% / 4 = 5 6/12 x 4
= 2% = 5.5 x 4
= 0.02 = 22

n
F=P 1 i
22
5, 500 1 0.02
22
5, 500 1.02
5,500 1.545980
P8,502.89

2. Accumulate P8,400 for 2 years at 7% converted monthly.


Given: P = P8,400 r = 7%, m = 12 t = 2 years

Solution:

i = r/m n=txm
= 7% / 12 = 2 x 12
= .005833 = 24

n
F=P 1 i
24
8, 400 1 0.005833
24
8, 400 1.005833
8,400 1.149806
P9,658.37
3. What sum of money will be required to settle a loan of P8,700 for 8 years and 6 months
at a rate of 9% compounded quarterly?

Given: P = P8,700 r = 9%, m = 4 t = 8 years and 6 months

Solution:

i = r/m n=txm
= 9% /4 = 8 6/12 x 4
= 2.25% = 8.5 x 4
=0.0225 = 34

n
F=P 1 i
34
8, 700 1 0.0225
34
5, 500 1.0225
5,500 2.130849
P18,538.39

4. Accumulate P15,400 for 5 years and 8 months at 5 ½% compounded semi – annually.

Given: P = P15,400 r = 5 ½%, m = 2 t = 5yrs and 8 mos.

Solution:

i = r/m n=txm
= 5 ½% /2 = (5 8/12) x 2
= 2.75% = (68 /12) x 2
=0.0275 = (17/3) x 2 = 34/3

n
F=P 1 i
34 / 3
15, 400 1 0.0275
34 / 3
15, 400 1.0275
15,400 1.359964
P20,943.44
5. If you deposit P5000 into an account paying 6% annual interest compounded monthly,
how long until there is P8000 in the account?

Given: F = P8,000 r = 6%, m = 12 P = P5,000

12 t
0.06
8000 5000 1
12
12 t
8000 5000 1 0.005
12 t
8000 5000 1.005
12 t
8000 5000 1.005
5000 5000
1.6 1.00512t

Take the logarithmic of both sides

log1.6 = log1.00512t
log1.6 = 12t log1.005
log1.6 12t log1.005
=
log1.005 log1.005
log1.6
= 12t
log1.005
94.23553232 12t
94.23553232 12t
12 12
t 7.9

6. Find the rate compounded quarterly if P745 accumulates to P786 in 3 years and 9
months.

Given: P = P745 F = 786 t = 3yrs and 9mos. m=4

Solution:
1
F n
r m 1 100%
p 1
4 1.055033557 15 1 100%
1
786 3.75 4
4 1.00357789 1 100%
4 1 100%
745
4 .00357789 100%
1
786 15 0.0143 100%
4 1 100%
745 1.43%

Practice Problems

Now it is your turn to try a few practice problems on your own.

Problem 1: If you deposit P4500 at 5% annual interest compounded quarterly, how muc
h money will be in the account after 10 years?

Problem 2: If you deposit P4000 into an account paying 9% annual interest compounde
d monthly, how long until there is P10000 in the account?

Problem3: If you deposit $2500 into an account paying 11% annual interest compounde
d quarterly, how long until there is P4500 in the account?

Problem4: How much money would you need to deposit today at 5% annual interest co
mpounded monthly to have P20000 in the account after 9 years?

Problem 5: If you deposit P6000 into an account paying 6.5% annual interest compound
ed quarterly, how long until there is P12600 in the account?
WEEK 15
Simple Annuities

Annuity is a series of periodic payment (usually equal) made at regular intervals of


time. Installment payments, monthly rentals, and life insurance premiums are familiar
examples of annuities.

The period of time between consecutive payments is called payment interval (pi).
Term of annuity is the time from the beginning of the first payment interval to the end of
the last payment interval.

The interval maybe of any convenient length like monthly (m=12), quarterly (m=4),
semi-annually (m=2), and annually (m=1).

Types of Annuities

A. Annuity Certain is an annuity payable for a definite duration not dependent on


some outside contingency. It means that this annuity begins and ends on a
definite or fixed date. Thus, monthly payment on a car from an annuity certain
because the payment start on a fixed date and continue until the require number
of payments has been made.

B. Annuity Uncertain or Contingent Annuity is an annuity payable for an indefinite


duration in which the beginning or the termination is depend on some certain
event. It means that this annuity’s first or last payment, or both, depends upon
some events. Thus pensions and life insurance policies are examples of annuity
uncertain.

Kinds of Annuity Certain

I. Simple Annuity an annuity whose interest conversion period (m) is equal or the
same as the payment interval (pi). [m = pi ]

Page 1 of 19
II. General Annuity an annuity whose interest conversion period (m) is unequal or
not the same as the payment interval (pi). [ m ≠ pi ]

Classification of Simple Annuity

1. Ordinary Annuity (Ao) is annuity in which the periodic payment (R) is made at
the end of each payment interval.

2. Annuity Due (A-due) is an annuity in which the periodic payment (R) is made at
the beginning of each payment interval.

3. Deferred Annuity (A-def) is an annuity in which the periodic payment is not


made at the beginning nor at the end of each payment interval, but some later
date.

The following notation will be used extensively throughout the discussion of annuities.

S = sum or amount of annuity


A = present value of annuity
Rs = periodic payment of the sum
t = term of annuity
r = rate of annuity
n = number of conversion period for the whole term (t x m)
m = number of conversion period per year
i = interest per conversion period (r÷m)
pi = payment interval
Ao = Ordinary Annuity
A – due = Annuity Due
A – def. = Deferred Annuity

Ordinary Annuity (Ao) Amount and Present Value


The amount of an annuity (S) is the total of all periodic payments at the end of the
term, while the present value (A) of annuity is the total of the present value of all the
payments of the annuity.

Page 2 of 19
Let’s us work on the following examples to illustrates the application above.

Illustrative Examples
1. Find the amount and present value of P1,500 payable every three months for 6 years
and 6 months. If money is worth 6%, m = 4.
Given: R = P1,500 r = 6% , m=4 t =6 yrs & 6 mos.
Solution: i = 6% ÷ 4 = 1 ½ = 0.015 n = 6½ x 4 = 26

26
1.015 1
1+i
n
1 = 1500
S = Rs 0.015
i
1.4727095344 1
6.5 4 = 1500
6% 0.015
1 1
4 0.4727095344
1500 = 1500
6% 0.015
4
26
0.06 = 1500 31.51396896
1 1
4
= 1500
0.06 = P 47, 270.95
4
26
1 0.015 1
= 1500
0.015

26
-n
1 1 0.015
1 1+i 1500
A = Ra 0.015
i 26
1 1.015
-6.5 4 1500
6% 0.015
1 1
4 1 0.6790205242
1500 1500
6% 0.015
4 0.3209794758
1500
26 0.015
0.06
1 1
4
1500 1500 21.39863172
0.06
4
P 32,097.95

Page 3 of 19
2. A man deposits P12,200 every end of 6 months in an account paying 5½%
compounded semi – annually. What amount is in the account at the end of 9
years and 6 months?
Given: Rs = P12,200 r = 5½%, m = 2 t = 9yrs & 6mos.
Solution: i = 5½% ÷ 2 = 2¾% = 0.0275 n = 9½ x 2 = 19

n
1+i 1
S = Rs
i
19
1 0.0275 1
12, 200
0.0275
19
1.0275 1
= 12, 200
0.0275
1.6743829015 1
= 12, 200
0.0275
0.6743829015
= 12, 200
0.0275
= 12, 200 24.5230146
= P 299,180.78

3. A home video entertainment set is offered for sale P18,000 down payment and
P1,800 every three months for the balance, for 18 months. If interest is to be
computed at 10% converted quarterly, what is the cash price equivalent of the
set?

Given: Dp = P18,000 r = 10% t =18 months


Ra = P 1,800 m=4

Solution:
i = 10% ÷ 4 = 2.5% = 0.025 n = (18/12)x4 = 1.5 x 4 = 6

Page 4 of 19
-n
1 1+i
CE = Dp + Ra
i
-6
1 1 0.025
P18,000 + 1, 800
0.025
6
1 1.025
18,000 + 1, 800
0.025
1 0.862296866
18,000 + 1, 800
0.025
0.137703134
18,000 + 1, 800
0.025
18,000 + 1, 800 5.50812536
18,000 + 9,914.63
P 27,914.63

Periodic Payment (R) of an Ordinary Annuity

Usually in business the amount or present value of annuity is known and the
periodic payment (R) may be computed by the following formula.
S A
Rs = n
and Ra = -n
1 i 1 1 1+i
i i

Let us work on the following examples to illustrate the application of the above.

Illustrative Examples

1. How much monthly deposit must be made for 5 years and 5 months in order to
accumulate P120,000 at 15% compounded monthly?

Given: S = P120,000 r = 15%, m = 12 t = 5yrs. & 5 mos.


Solution:
i = 15% ÷ 12 = 0.0125 n = (5 5/12) x 12 = 65

Page 5 of 19
S 120,000
Rs = =
1 i
n
1 2.2422140657 1
i 0.0125
120,000
120,000 =
= 65
1.2422140657
1 0.0125 1 0.0125
0.0125 120,000
=
120,000 99.377125256
=
1.0125
65
1 120,000
=
0.0125 99.377125256
= P 1,207.52

2. What sum will be paid at the end of each quarter for 6 years and 6 months, if the
present value is P50,500 and interest is paid at 10% compounded quarterly?
Given: S = P50,500 r = 10%, m = 4 t = 6yrs. & 6mos.
Solution:
i = 10% ÷ 4 = 0.025 n = (6 6/12) x 4 = 26

A
Ra = -n
1 1+i
i
50, 500
-26
1- 1 0.025
0.025
50, 500
-26
1- 1.025
0.025
50, 500
1- 0.5262347214
0.025
50, 500
0.4737652786
0.025
50, 500
18.950611144

2, 664.82

Page 6 of 19
3. Dino wants to buy a car worth P740,000. He can pay 40% of the prices as down
payment and the balance payable every end of the month for 60 months, how much
must he pay monthly at 15% compounded monthly?
Given: Dp = P740,000 x 405 = P444,000 r = 15%, m = 12 t =60 mos.
Solution:
i = 15% ÷ 12 = 0.125 n = (60/12) x 12 = 60

A
Ra = -n
1 1+i
i
444, 00
-60
1- 1 0.0125
0.0125
444, 000
-60
1- 1.0125
0.0125
444, 00
1- 0.4745676026
0.0125
444, 000
0.5254323974
0.0125
444, 000
42.034591792

P 10, 562.73

Finding the Term of Annuity

When the number of payments or deposits is not exactly equivalent to the original
amount or present value, there are various procedures in finding the term of annuity.
In this module, we shall use the procedure where the small payment/deposit or
concluding payment/deposit (c.p) and (c.d) can be made one period after the last
regular payment or deposit. But sometimes the concluding payment or deposit is not

Page 7 of 19
required because the interest after the last regular payment or deposit is equal to or more
than the balance needed.

Let us work on the following examples to illustrate the above.

1. Ella borrows P20,000 with interest at 5½% compounded quarterly. He agrees to pay
P850 at the end of each quarter. How long must he pay? What is the size if the concluding
payment?
Given: A = P20,000 Ra = P850 r = 5½%, m = 4
Solution:
i = 5½% ÷4 = 0.01375

-n
1 1+i
A = Ra
i
-n
1 1+0.01375
20,000 = 850
0.01375
n
20, 000 0.01375 850 1 1.01375
n
275 850 1 1.01375
850 850
n
0.3235294118 1 1.01375
n
0.3235294118 1 1.01375
n
0.676470588 1.01375
n
0.676470588 1.01375
log 0.676470588 n log 1.01375
log 0.676470588
-n=
log 1.01375
log 0.676470588
-n=
log 1.01375
-n= 28.62162
n = 28.62162
n 29

Page 8 of 19
Determine the concluding payment
Step 1. Determine the amount one period after the last regular payment by the
use of :
n
1+i 1
S = Rs
i

Step 2. Accumulate the present value one period after the last regular payment
by the use of:
n
F = P 1+I

Step 3. Subtract the result of 1 from the result of 2 get the concluding payment.
Cp = F - S

Solution:
Step 1.
n
1+i 1
S = Rs
i
29
1 0.01375 1
850
0.01375
29
1.01375 1
850
0.01375
1.4859190459 1
850
0.01375
0.4859190459
850
0.01375
850 35.339566975
P30, 038.63

Step 2.
n
F = P 1+I

29
F = 20,000 1 0.01375
29
= 20,000 1.01375
= 20,000 1.4859190459
= 29, 718.38

Page 9 of 19
Step 3.
Cp = F - S
= 29,718.38 - 30,038.63
= - P 320.25
2. Find the time and the number of semi – annual payments of P850 each and the
concluding deposit, which must be made to accumulate P18,500, if money earns interest
at 7% compounded semi –annually.
Given: Rs = P850 S = P18,500 r = 7%, m = 2
Solution:
i = 7% ÷ 4 = 3½% = 0.035

n
1+i 1
S = Rs
i
n
1 0.035 1
18,500 850
0.035
n
1.035 1
18,500 850
0.035
n
1.035 1
0.035 18,500 850 0.035
0.035
n
0.035 18,500 850 1.035 1
n
647.5 850 1.035 1
n
647.5 850 1.035 1
850 850
n
0.7617647059 1.035 1
n
0.7617647059 1 1.035
n
1.7617647059 1.035
log1.7617647059 n log1.035
log1.7617647059
n=
log1.035
n = 16.4619
n 17

Page 10 of 19
Finding the Interest Rate of an Annuity

If the interest rate is to determined, we use linear interpolation and by formula.

Let us work on the following examples to show this is being done.

Illustrative Examples
1. AMA company is planning to create a fund by depositing P1,520 every quarter for 8
years in a bank that promises to pay a fixed rate over the entire term. If the amount in
the fund at the end of the term is P60,000. Find the rate converted quarterly.

Given: S = P60,000 R = P1,520 t = 8years


m=4 r=?

To find the rate (r): First, solve for I using quadratic formula.

-b b2 - 4ac
i=
2a
Where:
Type I: S and R given Type II: A and R are given
A n2 – 1 n2 – 1
B - 6 (n – 1) 6 (n – 1)
C 12 (1 – nR/s) 12 (1 – nR/A)

Note:
a. Type I problem, use only the minus ( - ) sign of the plus or minus (±) in the
quadratic formula.
b. Type II problems, use only the plus ( + ) sign.
c. After solving for i : find the r: r = i x m

Page 11 of 19
Type I
Solution:
n=txm a = n2 – 1 b = -6 (n – 1 ) c = 12 ( 1 – nR/s)
=8x4 = 322 – 1 = -6 (32 – 1) = 12 (1 – 32x1520/60000)
= 32 =1024 -1 = -6 (31) = 2.27
= 1023 = -186
Substitute:

-b b2 - 4ac
i=
2a
2
- 186 - 186 - 4 1025 2.27
i=
2 1023
186 159.08
i=
2046
i = 0.13157
r = .01357 x 4
r = 5.26%

2. Abenson offers a TV set for P45,000 cash. The set is also available on installment basis
at P15,000 down – payment and monthly installment of P2,240 for 15 months. If interest is
compounded monthly, find the rate used.

Given: CV = P45,000
Less Dp = 15,000
A = P30,000
R = P2,240
t = 15 months
r=?

Page 12 of 19
Type II

Solution:
n = 15

a = (n2 – 1) b = 6 (n+1) c = 12 (1 – 15x2240/30000)


= (152 – 1) = 6 (15 + 1) = -1.44
= (225 -1) = 6 (16)
= 224 = 96

Solution:

- 96 + 962 - 4 224 1.44


i=
2 224
96 102.50
i=
448
i = 0.014509
r = 0.014509 x 12 x 100%
r = 17.41%

Page 13 of 19
WEEK 15
GENERAL ANNUITIES

To solve the problems under general annuity, we should get the payment
inter p and the compounding period c to coincide in order to facilitate out
computations and apply the principles and formulas of simple annuity.

We shall consider the following annuities:


1. Genera Ordinary Annuity where the first payment is made at the beginning of
every payment interval.
2. General Annuity Due where the first payment is made at the beginning of
every payment interval.
3. Perpetuities a series of periodic payment, which are to run indefinitely.

General Ordinary Annuity G(Ao)


A. Present Value (A)
To calculate the present value (A), we consider this formula:
-n
1- 1+i
A=R i
k
1+i -1
i

Where:
A = present value
R = periodic payment
n = total number of compounding periods for the whole term (txm)
m = number of compounding period for one year
i = rate per compounding period (r÷m)
c = number of months in compounding period
p = number of months in a payment interval
k = the number of compounding periods in a given payment interval k = (p÷c)

Page 14 of 19
Note:
When the payment interval (n) is compounded:
Monthly p = 1 month
Quarterly p = 3 months
Semi – annually p = 6 months
Annually p = 12 months

Illustrative Examples:

1. Find the present value of an annuity P 5,500 every end of three months for 10
years, if interest rate is 4% compounded annually.
Given: R = P 5,500 t = 10 years
r = 4% p = 3 months
m=1 c = 12 months
Solution
i =r÷m n=txm k=p÷c
= 4% ÷ 1 = 10 x 1 = 3 ÷ 12
= 4% or .04 = 10 = ¼ or .25
*use this if you have a table of Annuity

-10
1- 1+0.04
A = 5,500 0.04
0.25
1+0.04 -1
0.04
-10
1- 1.04
= 5,500 0.04
0.25
1.04 -1
0.04
8.110896
= 5,500
0.246335
= 5,500 32.926283
= P181, 094.56

Page 15 of 19
2. If interest rate i6 6% compounded quarterly, what is the present value of P7,700
payable at the end of each six months for 8 years?

Given: R = P7,700 m=4 p = 6 months


r = 6% t=8 c = 3 months

Solutions:
i = r ÷m n=txm k=p÷c
= 6% ÷ 4 =8x4 =6÷3
= 1½% = 32 =2 *use this if you have a table of Annuity

= 0.015

-32
1- 1+0.015
A = 7,700 0.015
2
1+0.015 -1
0.015
-32
1- 1.015
= 7,700 0.015
2
1.015 -1
0.015
25.267139
= 7,700
2.015
= 7,700 12.53952
= P96,554.33

3. Find the cash equivalent of sala set with a down payment of P12,000 and P1,450 for
the balance payable every end of the month for 30 months with interest rate at 8%
compounded quarterly.
Given: Dp = P12,000 r = 8% t = 30 mos. p = 12 mos.
R = P 1,450 m =4 c = 3 mos.

Page 16 of 19
Solution:
i = r ÷m n = (30/12)x 4 k=p÷c
= 8% ÷ 4 = 2.5 x 4 = 12 ÷3
= 2% = 10 = 4 *use this if you have a table of Annuity
= 0.02

-10
1- 1+0.02
CE =12,000 + 1,450 0.02
4
1+0.02 -1
0.02
-10
1- 1+0.02
= 12,000 + 1,450 0.02
4
1+0.02 -1
0.02
-10
1- 1.02
= 12,000 + 1,450 0.02
4
1.02 -1
0.02
8.982585
= 12,000 + 1,450
4.121608
= 12,000 + 1,450 2.179388
= 12,000 + 3,160.11
= P 15,160.11

Periodic payment of the Present Value of G(Ao)

Periodic payment (R) can be computed using this formula.


k
1+ i 1
R =A i
n
1 1+ i
i

Page 17 of 19
Illustrative Examples:
1. Find the periodic payment payable every six months for 10 years, if the present value
is P125,000 with interest rate at 6% compounded annually.
Given: A = P125,000 m =1 p =6 months
r = 6% t = 10 years c = 12 months
Solution:
i=r÷m n=txm k=p÷c
= 6% ÷ 1 = 10 x 1 = 6 ÷ 12
= 6% = 10 = 1/2
= 0.06

0.5
1+ 0.06 1
R =125,000 0.06
10
1 1+ 0.06
0.06
0.5
1.06 1
=125,000 0.06
10
1 1.06
0.06
.492717
=125,000
7.360087
=125,000 0.066944
= P8,368.06
2. What will be the size of the payments, if the interest rate is 5% compounded quarterly
on a P87,000 loan payable every end of each year for 12 years.
Given: A = P87,000 m=4 p = 12 months
r = 5% t = 12 years c = 3 months

Solution:
i=r÷m n=txm k=p÷c
= 5% ÷ 4 = 12 x 4 = 12 ÷ 3
= 1¼% = 48 =4
= .0125

Page 18 of 19
4
1+ 0.0125 1
R = 87,000 0.0125
10
1 1+ 0.0125
0.0125
4
1.0125 1
=87,000 0.0125
10
1 1.0125
0.0125
4.075627
= 87,000
35.931481
=87,000 0.1134277
= P 9,868.21

Page 19 of 19
WEEK 15

Basic concept of Mortgage, Stocks and Bonds

This module tackles topics on Stocks and Bonds.

Course Module Objectives:


At the end of this module, the learner should be able to:
1. Interpret the theory of efficient markets
2. Illustrate business and consumer loans
3. Distinguish between business and consumer loans
4. Understand the concepts of Stocks and Bonds
5. Solve problems involving Mortgage, Stocks and Bonds

Given your knowledge on stocks and bonds, discuss in class how you can profit on the
buying and selling of these financial options.

Theory of Efficient Markets

Before we discuss the theory of efficient markets, let us check some definitions.

Fundamental Analysis – this is the analysis of different public information about a stock.

Technical Analysis – this refers to the analysis of patterns and historical data (prices) of stocks.

The theory of efficient market was developed by Eugene Fama in 1970. This
theory states that the stock prices reflect all available information about the stock
(including historical information). This hypothesizes that the stock prices are accurate.
Thus the slogan “Trust Market Prices” was coined to support the theory. This concept
assures the public that no overvaluing or undervaluing is happening to stock prices.

The question now is if the theory is true. In some market, like in the Philippines,
some investors might argue that there is a pattern in the market prices. Some investors
argue that the patterns can be studied to be able to predict the market prices in the
future. If this is true, then the risk of investing in stocks will be lesser than what we have
now. This is an issue which is open for discussion.

Create two teams in class. Have one team in favor of the theory of efficient
markets and the other on the other side. Discuss the validity of the theory and make
sure that points are thrown to support your claims. Your teacher will choose which side
presented a stronger argument.

Use the following arguments:

We can beat the market.

We cannot beat the market.

Basic Loans

Suppose your parents wanted to buy a house for your growing family, they went to
several banks and financial institutions to check their options. They decided to apply for
a home loan from a bank.

Definition of Terms:

Business Loan – money lent to fund for business transactions.

Consumer Loan – money lent to fund personal or family expenses.

Collateral – assets used to secure a loan.

Term of the Loan – time to pay the entire loan.

Consumer loans on the other hand is used for personal or family expenses. These loans
do not usually require collaterals as the money lent for consumer loans are generally
smaller. Moreover, the interest rates are normally lower for consumer loans and the term
are longer.

Business loans can be used to start a business or to expand businesses. Business loans
often require collaterals in forms of real estate or other investments.

Solving Problems on Business and Consumer Loans

Definition of terms:
Amortization Method – method of paying for the loan on installment basis.

Mortgage – loan, secured by a collateral, payed at specified terms.

Chattel Mortgage – mortgage on movable property (i.e. vehicles).

Collateral – assets used to secure a loan.

Outstanding Balance – any remaining debt at a specified time.

Example 1. Mr. Garcia borrowed Php 1,000,000.00 for the expansion of his business. The
effective rate of interest is 7%. The loan is to be repaid in full after one year. How much is
to be paid after one year?

Solution:

This is treated as a simple interest with term = 1.

F = P(1+i)n =1,000,000 (1+0.07)=1,070,000.00

Example 2. Ms. Samson borrowed Php 1,200,000.00 for the purchase of a car. If the
monthly payment is Php 30,000.00, how much is the interest paid for a term of 5 years?

Solution:

Since the monthly payment is Php 30,000.00 for 5 years,

Total Payment=Php 30,000.00 ×12 mos/year× 5 years


Total Payment = Php 1,800,000.00
Interest = Total Payment-Money Borrowed
Interest = Php 1,800,000.00 - Php 1,200,000.00
Interest = Php 600,000.00

Example 3. If your house was bought for Php 3,000,000.00 and the bank required 20%
down payment, find the amount of the mortgage you took on the bank for your house.

Solution:
First thing to do is to compute for the down payment for the house.

Down payment=Php 3,000,000.00 × 0.20


Down payment=Php 600,000.00
Mortgage=Cash Value-Down payment
Mortgage=Php 3,000,000.00-Php 600,000.00
Mortgage=Php 2,400,000.00

In solving problems on basic loans, you need to use prior knowledge on interest
and annuities as the computations in basic loans will most often use formulas from
interests and annuities.

Stocks

Some corporations may raise money for their expansion by issuing stocks. Stocks are
shares in the ownership of the company. Owners of stocks may be considered as part
owners of the company. There are two types of stocks: common stock and preferred
stock. Both will receive dividends or share of earnings of the company. Dividends are
paid first to preferred shareholders.

Stocks can be bought or sold at its current price called the market value. When a person
buys some share, the person receives a certificate with the corporation XYZ name, owner
XYZ name, number of shares and par value per share.

Bonds

Bonds are interest bearing security which promises to pay amount of money on a certain
maturity date as stated in the bond certificate. Unlike the stockholders, bondholders are
lenders to the institution which may be a government or private company. Some bond
issuers are the national government, government agencies, government owned and
controlled corporations, Non-bank Corporation is, banks and multilateral agencies.

Bondholders do not vote in the institution XYZ annual meeting but the first to claim in the
institution XYZs earnings. On the maturity date, the bondholders will receive the face
amount of the bond. Aside from the face amount due on the maturity date, the
bondholders may receive coupons (payments/interests), usually done semi-annually,
depending on the coupon rate stated in the bond certificate

Distinguished between stocks and bonds.

STOCKS BONDS
A form of equity financing or raising A form of debt financing, or raising money
money by allowing investors to be part by borrowing from the investors
owners of the company.
Stock prices vary every day. These prices Investors are guaranteed payments and
are reported in various media return of their money at the maturity date.
(newspaper, TV, Internet, etc.)
Investing in stocks involves some Uncertainty comes from the ability of the
uncertainty. Investors can earn if the stock bonds issuer to pay the bondholder. Bonds
prices increase, but they can lose money issued by the government pose less risk
If the stock prices decrease or worse, if the than those companies because the
company goes bankrupt. government has guaranteed funding
(taxes) from which it can pay its loans.
Higher risk but with possibility of higher Lower risk but lower yields
return.
Can be appropriate if the investments is Can be appropriate for retirees (because
for the long term (10 years or more). This of the guaranteed fixed income) or for
can allow investors to wait for stocks prices those who need the money soon
to increase if ever they go low. (because they cannot to take a chance
at the stocks market)

Definition of terms in relation to stocks

Stocks – share in the ownership of a company


Dividend – share in the company
Dividend Per Share – ratio of the dividends to the number of shares
Stock Market - a place where stocks can be bought or sold. The stock market in
the Philippines is governed by the Philippine Stock Exchange (PSE).
Market Value - the current price of a stock at which it can be sold.
Stock Yield Ratio - ratio of the annual dividend per share and the market value
per share. Also called current stock yield.
Par Value - the per share amount as stated on the company certificate. Unlike
market value, it is determined by the company and remains stable over time.
Illustrative Example

Example 1. A certain financial institution declared a P30, 000,000 dividend for the
common stocks. If there are total of 700,000 shares of common stock. How much is
dividend per share?

Given: Total Dividend = P30,000,000


Total Shares = 700,000

Find: Dividend per share


Total Dividend
Dividend per share =
Total Share
30, 000, 000
=
700, 000
= 42.86
Therefore, the dividend per share is P42.86

Example 2. A certain corporation declared a 5% dividend on a stock with a par value of


P500. Mr. Mendoza owns 300 shares of stock with a par value of P500. How much id the
dividend he received?

Given: Dividend Percentage = 5%


Number of Shares = 300

Find: Dividend per share

The Dividend per share is = Par Value Dividend Percentage


= 500 0.05
= P25
The total dividend is = Dividend Per Share Number of share
= 25 300
= P7500

In summary,
Dividend = Dividend Percentage Par Value Number of share
= 0.05 500 300
= P7500
Thus, the dividend is P7,500.

Example 3. Corporation A, with current market value of P52, gave a dividend of P8 per
share for its common stock. Corporation B, with a current market value of P95, gave a
dividend of P12 per share. Use the stock yield ratio to measure how much dividends
shareholders are getting in relation to the amount invested.

Solution: Corporation A: Corporation B:

Given: Dividend per share = P8 Given: Dividend per share = P12


Market value = P52 Market value = P95

Find: Stock Yield Ratio Find: Stock Yield Ratio

Dividend per Share Dividend per Share


Stock Yield Ratio = Stock Yield Ratio =
Market Value Market Value
8 12
= =
52 95
= 0.1538 = 0.1263
= 15.38% = 12.63%

Corporation A has higher stock yield ratio than Corporation B. Thus, each peso would
earn you more if you invest in corporation A than in Corporation B. If all other things are
equal, then it is wise to invest in Corporation A.

Definition of terms in relation to Bonds.


Bond – interest – bearing security which promise to pay
(1) A stated amount of money on the maturity date, and
(2) Regular interest payments called coupons.

Classification of Bonds according to security provided for a bond

1. Registered Bonds – Bonds that can only be transferred from one owner to another
by proper endorsement and with the consent of the issuer. Thus, the owner is
protected against loss or theft.
2. Unregistered Bonds –Bonds that can be transferred
Coupon – periodic interest payments that the bondholder receives during the time
between purchase date and maturity date; usually received semi – annually
Coupon Rate – the rate per coupon payment period; denoted by r
Price of a Bond – the price of the bond at purchase time; denoted by P
Par Value of Face Value – the amount payable on the maturity date; denoted by
F.
If P = F, the bond Is purchased at par
If P < F, the bond is purchased at a discount
If P > F, the bond is purchased at premium
Term ( or Tenor ) of the Bond – fixed period of time (in years) at which the bond is
redeemable as stated in the bond certificate; number of years from time of
purchase to maturity date
Fair Price of a Bond – present value of all cash inflows to the bondholder
Redemption Value – The amount that will be paid on the redemption date
Redemption rate - the rate on the principal; it is used to find the redemption value.

Rate associated with bonds

1. Bond rate (Br) – the rate at which the bond pays interest on its par or face value.
2. Yield rate (Y) – the true overall rate of return that an investor receives on the
invested capital.
Notation:

FV = Par or Face Value of bond


RV = Redemption value of a bond
Br = Bond rate
Y = Yield rate
Rr = Redemption rate
Cp = Coupon payment
m = Conversion period per year
n = total number of interest per period for the whole term (txm)
b = bond rate
i = yield rate per interest period (Y/m)
Pp = Purchase price or value of the bond
Bp = Bond premium
Bd = Bond discount
MQ = market Price
BV = Book value

Illustrative Examples

Example 4. Determined the amount of the semi – annual coupon for a bond with a face
value of P300,000 that pays 10%, payable semi-annually for its coupons.
Given: Far Value F = P300,000
Coupon Rate r = 10%

Find: Amount of the semi-annual Coupon

Annual coupon amount = Far Value Coupon Rate x 1


= 300, 000 .10
= 30,000

1
Semi- Annual coupon amount = 30,000
2
= 30,000 0.5
= 15, 000

Thus, the amount of the semi-annual coupon is P15,000.

The coupon rate is used only for computing the coupon amount, usually paid se-
annually. It is not the rate at which money grows. Instead current market condition are
reflected by the market rate, and is used to compute the present value of future
payments.

Example 5. A P50,000, 10% bond with quarterly coupons is redeemed at 115% at the end
of the 10 years, Find the coupon payments and redemption values. (at premium)

Given: Fv = P50,000 Rr = 115% Br = 10%, m = 4

Solution:
b = 10% 4 =.025 Rr = 1.15

Cp = FV x b Rv = FV x Rr
= 50,000 x .025 = 50,000 x 1.15
= P 1,250 = P 57,500
Example 6. A P20,000, 11% bond with semi-annual coupons is redeemed at 95% at the
end of 15 years. Find the coupon payment and redemption value. (at discount)

Given: Fv = P20,000 Rr = 95% Br = 11%, m = 2

Solution:
b = 11% 2 =.055 Rr = .95

Cp = FV x b Rv = FV x Rr
= 20,000 x .055 = 20,000 x .95
= P 1,100 = P 19,000

Example 7. A P40,000, 14% bond with quarterly coupons is redeemed at 100% at the end
of 15 years. Find the coupon payments and redemption value. (at par)

Given: Fv = P40,000 Rr = 100% Br = 14%, m = 4

Solution:
b = 14% 4 =.035 Rr = 1.00

Cp = FV x b Rv = FV x Rr
= 40,000 x .035 = 40,000 x 1.00
= P 1,400 = P 40,000
Example 8. Find the purchase price of the bond valued at P50,000 with interest at 5 ½%
payable quarterly at redeemable at 96% in 15 years, if the bond yields 6%, m = 4 on the
investment.

Given: Fv = P50,000 Rr = 96% Br = 14%,


m=4 Y = 6% t = 15 years

Solution:
1
b=5 % 4 =.01375 Rr = 1.00
2
n = 15 x 4 =60 i = 6% 4 = .015

Cp = FV x b Rv = FV x Rr
= 50,000 x .01375 = 50,000 x .96
= P 687.50 = P 48,000

-n
-n 1- 1+i
Pp =RV 1+i + Cp
i
-60
-60 1- 1+0.015
=48000 1+0.015 + 687.50
0.015
=48000 .409296 + 687.50 39.380269
=19, 646.21 + 27, 073.93
= P 46,720.14

Bd = RV - Pp
= 48,000 - 46,720.14
= P1,279.86
General Mathematics
1
Propositional Logic; Syllogisms and Fallacies – Part 001

Propositional Logic; Syllogisms and Fallacies -


Part 001
This module tackles topics on Propositional Logic, Syllogisms and Fallacies.
Course Module Objectives:
At the end of this module, the learner should be able to:
1. Illustrate a proposition
2. Symbolize propositions
3. Distinguish between simple and compound propositions
4. Perform different types of operations on propositions
“Everyone needs to study logic.” Use this statement as a point of discussion in
class. Divide your class in two groups. One will be in favor of the statement
and one against it. Create arguments that will support your group’s position
on the statement by brainstorming within your group. Use a debate as a
medium to discuss your arguments and let your teacher be the judge on who
wins the debate.

Proposition
The logic is a system based on propositions. One needs to understand what
propositions are and identify statements as proposition or not to be able to
justify arguments using logic.
Definition of terms:
1. Logic is the systematic study of the form/s of arguments. Logic is used to
prove or disprove arguments.
2. An argument is a series of statements typically used to persuade someone
of something or to present reasons for accepting a conclusion.
3. A statement is a meaningful declarative sentence.
4. A proposition is a statement that is either true or false (not both). We say
that the truth value of a proposition is either true (T) or false (F).
To be able to clearly and efficiently use logic for proof and disproof, one
needs to be able to understand propositions.
Reminder: A propositions needs to have a truth value. A sentence without a
truth value is not a proposition but a mere statement. Note that a proposition
should only have one value. It should only be either true or false and not
both. In logic, statements can be in words or numerical format.
Examples
1. “Mammals are bigger than insects.”

Course Module
This is a statement as it states a meaningful sentence. This is also a
proposition as it has a truth value which is true. No insects are bigger
than any mammal.
2. “500 < 300”.
This is a statement. It is a numerical statement. This is also a proposition
as it has a truth value which is false. This is read as “five hundred is less
than three hundred” which in mathematical perspective is false as five
hundred is not less than three hundred but is the opposite which is
greater.
3. “𝑦 > 5”.
This is a statement as it is a mathematical statement. This is not a
proposition. Although it could have a truth value, one cannot determine
its truth value unless an exact numerical value is given to 𝑦. Example, if
𝑦 = 1, the statement is false. If 𝑦 = 10, then the statement is true.
4. “I will get a perfect score in the Logic exam.”
This is a declarative sentence. Although the truth value of the statement
can only be known after the results of the exam is given, we know that the
value can only be either true or false. If I got 100 percent of the questions
correct, then the value is true. Otherwise, the value is false.
For a declarative sentence to be a proposition, it is not necessary that its
truth value be immediately known.
5. “Please do not fall asleep.”
Although the example is a sentence, it is not a statement. This is an
imperative sentence and not declarative which means it cannot be a
statement. Moreover, since it is not a statement, it cannot be a
proposition nor can it have a truth value.

Symbolizing Propositions
In the study of logic, propositions are commonly symbolized to make
computations easier.
Propositions are normally symbolized using a small letter.
Example:
𝑝: 𝑀𝑎𝑚𝑚𝑎𝑙𝑠 𝑎𝑟𝑒 𝑏𝑖𝑔𝑔𝑒𝑟 𝑡ℎ𝑎𝑛 𝑖𝑛𝑠𝑒𝑐𝑡𝑠.
This can be read as “p is the proposition “Mammals are bigger then insects.””.

Simple and Compound Propositions


A simple proposition is a statement which only states one meaning. It cannot
be broken down into other component propositions.
A compound proposition is a proposition formed from simple propositions
using logical operators or some combination of logical operators.
Examples:
Assume the p and q are propositions:
𝑛𝑜𝑡 𝑝 (could also be ~ 𝑝 where ‘~’ is the not operator)
𝑝 𝑎𝑛𝑑 𝑞 (could also be 𝑝 ∧ 𝑞 where ∧ is the ‘and’ operator)
𝑝 𝑜𝑟 𝑞 (could also be 𝑝 ∨ 𝑞 where ∨ is the ‘or’ operator)
General Mathematics
3
Propositional Logic; Syllogisms and Fallacies – Part 001

𝑖𝑓 𝑝 𝑡ℎ𝑒𝑛 𝑞 (could also be 𝑝 → 𝑞 where → is the ‘if-then’ operator; this can


also be read as ‘p implies q’)
Simple propositions are also called atomic propositions because they are
building block of propositional logic. Remember that compound propositions
are composed of two or more simple propositions.

Operations on Propositions

Logical Operators

The previous section introduced you to several logical operators that you will
be seeing and using throughout the remainder of this module. These logical
operators can be used to populate a truth table for the determination of the
truth value of simple and compound propositions.

Truth Table

In logic, given a proposition, a truth table shows all the possible truth values
of a proposition.
Example: (for a simple proposition)
The truth table is

A truth table is useful if we need to show the possible truth values of


compound propositions.
Example: (for compound propositions with p and q as component
propositions)
First determine the number of possible combinations of the simple
propositions. For 2 component propositions, the number of combinations is
4 (given by the equation “𝑁 = 2𝑛 ” where N is the number of combinations
and n is the number of component propositions.
It is useful to remember the truth values for each combination below as this
is the basis for most logic computations.

p q ~p ~q 𝑝∧𝑞 𝑝∨𝑞 𝑝→𝑞

Course Module
T T F F T T T

T F F T F T F

F T T F F T T

F F T T F F T

Examples:
𝑝: 𝑇ℎ𝑒 𝑡𝑖𝑛𝑖𝑘𝑙𝑖𝑛𝑔 𝑖𝑠 𝑡ℎ𝑒 𝑚𝑜𝑠𝑡 𝑑𝑖𝑓𝑓𝑢𝑐𝑙𝑡 𝑑𝑎𝑛𝑐𝑒.
𝑞: 𝐸𝑣𝑒𝑟𝑦𝑜𝑛𝑒 𝑖𝑛 𝑉𝑖𝑠𝑎𝑦𝑎𝑠 𝑠𝑝𝑒𝑎𝑘𝑠 𝐶𝑒𝑏𝑢𝑎𝑛𝑜.
𝑟: 𝐴𝑛𝑔𝑒𝑙𝑠 𝑒𝑥𝑖𝑠𝑡.
𝑠: 𝜋 > 3
1. ~𝑝: 𝑇ℎ𝑒 𝑡𝑖𝑛𝑖𝑘𝑙𝑖𝑛𝑔 𝑖𝑠 𝑛𝑜𝑡 𝑡ℎ𝑒 𝑚𝑜𝑠𝑡 𝑑𝑖𝑓𝑓𝑖𝑐𝑢𝑙𝑡 𝑑𝑎𝑛𝑐𝑒.
2. ~𝑞: 𝑁𝑜𝑡 𝑒𝑣𝑒𝑟𝑦𝑜𝑛𝑒 𝑖𝑛 𝑉𝑖𝑠𝑎𝑦𝑎𝑠 𝑠𝑝𝑒𝑎𝑘𝑠 𝐶𝑒𝑏𝑢𝑎𝑛𝑜.
3. 𝑝 ∧ 𝑞: 𝑇ℎ𝑒 𝑡𝑖𝑛𝑖𝑘𝑙𝑖𝑛𝑔 𝑖𝑠 𝑡ℎ𝑒 𝑚𝑜𝑠𝑡 𝑑𝑖𝑓𝑓𝑖𝑐𝑢𝑙𝑡 𝑑𝑎𝑛𝑐𝑒 𝑎𝑛𝑑 𝑒𝑣𝑒𝑟𝑦𝑜𝑛𝑒 𝑖𝑛
𝑉𝑖𝑠𝑎𝑦𝑎𝑠 𝑠𝑝𝑒𝑎𝑘𝑠 𝐶𝑒𝑏𝑢𝑎𝑛𝑜.
4. 𝑝 ∧ (~𝑞): 𝑇ℎ𝑒 𝑡𝑖𝑛𝑖𝑘𝑙𝑖𝑛𝑔 𝑖𝑠 𝑛𝑜𝑡 𝑡ℎ𝑒 𝑚𝑜𝑠𝑡 𝑑𝑖𝑓𝑓𝑖𝑐𝑢𝑙𝑡 𝑑𝑎𝑛𝑐𝑒 𝑎𝑛𝑑
𝑛𝑜𝑡 𝑒𝑣𝑒𝑟𝑦𝑜𝑛𝑒 𝑖𝑛 𝑉𝑖𝑠𝑎𝑦𝑎𝑠 𝑠𝑝𝑒𝑎𝑘𝑠 𝐶𝑒𝑏𝑢𝑎𝑛𝑜.
5. (~𝑟) ∨ 𝑠: 𝐴𝑛𝑔𝑒𝑙𝑠 𝑑𝑜 𝑛𝑜𝑡 𝑒𝑥𝑖𝑠𝑡 𝑎𝑛𝑑 𝜋 > 3.

Computing for truth values

Consider the following propositions.


𝑠: 𝜋 > 3
𝑡: 2 𝑖𝑠 𝑝𝑟𝑖𝑚𝑒.
𝑢: 2 𝑖𝑠 𝑜𝑑𝑑.
1. Determine the truth value of (~𝑠) ∧ 𝑡
Column 4
1 2 3
Number
First Step Second Step
Row 𝑠 t
~s (~𝑠) ∧ 𝑡
Number
T F T F
1
T F F F
2
F T T T
3
F T F F
4
Since we are using two simple propositions, the number of possible
combinations is 4, hence the four rows in the table above. You start with
the first proposition which is s. Getting the negation of the proposition,
you have the possible truth value in column 2.
Going back to the truth value of s and t, we know that s is true as is
3.1416 which is greater than 3 and that t is likewise true as 2 is a prime
number. Looking at the truth values of s and t from the table, you are to
General Mathematics
5
Propositional Logic; Syllogisms and Fallacies – Part 001

choose Row 1. Hence, the truth value of the compound proposition (~𝑠) ∧
𝑡 is false.
2. Determine the value of the compound proposition ~(𝑠 ∧ 𝑡) ∨ 𝑢
Column 6
1 2 3 4 5
Number
Row s t u 𝑠∧𝑡 ~(𝑠 ∧ 𝑡) ~(𝑠 ∧ 𝑡) ∨ 𝑢
Number
T T T T F T
1
T T F T F F
2
T F T F T T
3
T F F F T T
4
F T T F T T
5
F T F F T T
6
F F T F T T
7
F F F F T T
8
Looking at the truth table of the compound proposition, the value for
almost all possible combinations of the truth values of s, t and u is true
except for Row . Hence, any truth value for s, t and u is true for the
compound proposition ~(𝑠 ∧ 𝑡) ∨ 𝑢.
Another solution:
For propositions whose values can be readily known, you opt to
compute based on the truth value already so you do not need to
compute for all possible combination of truth values. Computing for
the same given:
𝑠: 𝜋 > 3 This is True
𝑡: 2 𝑖𝑠 𝑝𝑟𝑖𝑚𝑒. This is True
𝑢: 2 𝑖𝑠 𝑜𝑑𝑑. This is False
Computing for ~ ( 𝑠 ∧ 𝑡 ) ∨ 𝑢
~ ( 𝑇 ∧ 𝑇 ) ∨ 𝐹
~ ( 𝑇 ) ∨ 𝐹
𝐹 ∨ 𝐹
𝐹

References
Albay, Eduard M., et al., (2016). General Mathematics. Makati City: Diwa
Learning Systems, Inc.

Course Module
General Mathematics
1
Propositional Logic; Syllogisms and Fallacies – Part 002

Propositional Logic; Syllogisms and Fallacies -


Part 002
This module tackles topics on Propositional Logic, Syllogisms and Fallacies.
Course Module Objectives:
At the end of this module, the learner should be able to:
1. Illustrate the different forms of conditional propositions
2. Illustrate different types of tautologies and fallacies
3. Determine the validity of categorical syllogisms
4. Establish the validity and falsity of real-life arguments using logical
propositions, syllogisms and fallacies.

Constructing Truth Tables


Let us discuss the procedures by looking at some examples.
Example 1:
Let p and q be propositions. Construct the truth table of the compound
proposition (𝑝 → 𝑞) ∧ (𝑞 → 𝑝).
The example has two proposition so the number of combination you need is
4.

Column 1 2 3 4 5
Number

Row Number p q 𝑝→𝑞 𝑞→𝑝 (𝑝 → 𝑞) ∧ (𝑞 → 𝑝)

1 T T T T T

2 T F F T F

3 F T T F F

4 F F T T T

Example 1 shows another type of conditional statement – the biconditional


𝑝 ↔ 𝑞 which has the truth table:

p q 𝑝↔𝑞

T T T

T F F

Course Module
F T F

F F T

Example 2:
Consider the compound proposition 𝑠: ((𝑝 → 𝑟) ∧ (𝑞 → 𝑟)) → ((𝑝 ∨ 𝑞) → 𝑟).
Since we have three propositions, the number of possible combinations is 8.
Column 1 2 3 4 5 6 7 8 9
Number

Row p q r 𝑝→𝑟 𝑝→𝑟 ((𝑝 → 𝑟) ∧ (𝑞 → 𝑟) 𝑝∨𝑞 (𝑝 ∨ 𝑞) → 𝑟 s


Number

1 T T T T T T T T T

2 T T F F F F T F T

3 T F T T T T T T T

4 T F F F T F T F T

5 F T T T T T T T T

6 F T F T F F T F T

7 F F T T T T F T T

8 F F F T T T F T T

Tautology and Contradiction


Definition:
1. Tautology – this is a proposition that is always true. (denoted by )
2. Contradiction – this is a proposition that is always false. (denoted by ∅)
Examples:
Let p and q be propositions. Determine the truth table for the following:
1. 𝑝∨𝜏
2. 𝑝∧∅
3. 𝑝 → (𝑝 ∨ 𝑞)
4. (𝑝 ∧ (~𝑞)) ∧ (𝑝 ∧ 𝑞)
Solutions:

1.

2.
General Mathematics
3
Propositional Logic; Syllogisms and Fallacies – Part 002

3.

4.

Valid Arguments and Fallacies


Definition of terms:
1. A lemma is a simple theorem used as an intermediate result in the proof
of another theorem.
2. A corollary is a proposition that follows directly from a theorem that has
been proven.
3. A conjecture is a statement whose truth value is unknown. Once it is
proven, it becomes a theorem.
4. An axiom is a basic assumption about mathematical structures that
needs no proof.
5. We can use a proof to demonstrate that a particular statement is true. A
proof consists of a sequence of statements that form an argument.
6. The steps that connect the statements in such a sequence are the rules of
inference.
7. Cases of incorrect reasoning are called fallacies.
8. A theorem is a statement that can be shown to be true.

Rules of Equivalence

Discuss in class the rules of equivalence given above. In most of the given
rules, mathematical properties are also parallel to them.
Course Module
Rules of Inference

• Rules of inference provide the justification of the steps used in a proof.


• One important rule is called modus ponens or the law of detachment.
It is based on the tautology
(p (p q)) q. We write it in the following way:
p
p q
____
q
Note: The two hypotheses p and p q are written in a column, and the
conclusion below a bar, where means “therefore”.

The general form of a rule of inference is:


p1
p2
.
.
.
pn
____
q
The rule states that if p1 and p2 and … and pn are all true, then q is true as
well.
These rules of inference can be used in any mathematical argument and do
not require any proof.

Propositional Form Standard Form

Rule of Simplification (𝑝 ∧ 𝑞) → 𝑝 𝑝∧𝑞


∴𝑝

Rule of Addition 𝑝 → (𝑝 ∨ 𝑞) 𝑝
∴𝑝∨𝑞

Rule of Conjunction (𝑝 ∧ 𝑞) → (𝑝 ∧ 𝑞) 𝑝
𝑞
∴𝑝∧𝑞

Modus Ponens ((𝑝 → 𝑞) ∧ 𝑝) → 𝑞 𝑝→𝑞


𝑝
∴𝑞

Modus Tollens ((𝑝 → 𝑞) ∧ (~𝑞)) → ~𝑝 𝑝→𝑞


~𝑞
∴ ~𝑝

Law of Syllogism ((𝑝 → 𝑞) ∧ (𝑞 → 𝑟)) → (𝑝 → 𝑟) 𝑝→𝑞


𝑞→𝑟
∴𝑝→𝑟
General Mathematics
5
Propositional Logic; Syllogisms and Fallacies – Part 002

Rule of Disjunctive ((𝑝 ∨ 𝑞) ∧ (~𝑝)) → 𝑞 𝑝∨𝑞


Syllogism ~𝑝
∴𝑞

Rule of Contradiction ((~𝑝) → 𝜙) → 𝑝 (~𝑝) → 𝜙


∴𝑝

Rule of Proof by Cases ((𝑝 → 𝑟) ∧ (𝑞 → 𝑟)) → ((𝑝 ∨ 𝑞) → 𝑟) 𝑝→𝑟


𝑞→𝑟
∴ (𝑝 ∨ 𝑞) → 𝑟

Arguments

Just like a rule of inference, an argument consists of one or more hypotheses


and a conclusion.
We say that an argument is valid, if whenever all its hypotheses are true, its
conclusion is also true.
However, if any hypothesis is false, even a valid argument can lead to an
incorrect conclusion.
Example 1:
If 101 is divisible by 3, then 1012 is divisible by 9. 101 is divisible by 3.
Consequently, 1012 is divisible by 9.”
Although the argument is valid, its conclusion is incorrect, because one of the
hypotheses is false (“101 is divisible by 3.”).
If in the above argument we replace 101 with 102, we could correctly
conclude that 1022 is divisible by 9.
Which rule of inference was used in the last argument?
p: “101 is divisible by 3.”
q: “1012 is divisible by 9.”
𝑝→𝑞
𝑝
∴𝑞
Modus Ponens was used.
Unfortunately, one of the hypotheses (p) is false.
Therefore, the conclusion q is not guaranteed to be correct.
Discuss the next example in class.
Example 2:

Course Module
“If it rains today, then we will not have a barbeque today. If we do not have a
barbeque today, then we will have a barbeque tomorrow.
Therefore, if it rains today, then we will have a barbeque tomorrow.”
Example 3:
“Gary is either intelligent or a good actor.”
“If Gary is intelligent, then he can count from 1 to 10.”
“Gary can only count from 1 to 2.”
“Therefore, Gary is a good actor.”
Symbolize the propositions to create the argument:
i: “Gary is intelligent.”
a: “Gary is a good actor.”
c: “Gary can count from 1 to 10.”
Proof that the argument is valid:
Statement Proof
Step 1: ~𝑐 Hypothesis
Step 2: 𝑖 → 𝑐 Hypothesis
Step 3: ~𝑖 Modus Tollens (Steps 1 & 2)
Step 4: 𝑎 ∨ 𝑖 Hypothesis
Step 5: 𝑎 Disjunctive Syllogism (Steps 3 & 4)
Conclusion: 𝑎 : “Gary is a good actor.”

References
Albay, Eduard M., et al., (2016). General Mathematics. Makati City: Diwa
Learning Systems, Inc.
General Mathematics
1
Methods of Proofs

Methods of Proofs
This module tackles topics on Propositional Logic, Syllogisms and Fallacies.
Course Module Objectives:
At the end of this module, the learner should be able to:
1. Illustrate the different methods of proof (direct and indirect) and disproof
(indirect and counterexample)
2. Justify mathematical and real-life statements using the different methods
of proof and disproof

Basic Idea of proofs


The goal of the proof is to show that the conclusion logically follows from the
given propositions (or premises).
As for the content of the proof, each proposition must be a valid assertion.
This means that the propositions must be based on a given statement (i.e. a
premise or a hypothesis), or they must follow from the premise via rules of
inference.
Example 1:
𝑝 → (𝑟 ∧ 𝑠)
~𝑟
∴ ~𝑝
We need to assume that all propositions above the line are true. These are
our premise or our hypotheses. The goal is to establish a logical sequence of
statements to arrive to the conclusion. We can only use rules of equivalence
and rules of inference for justification. We may summarize the steps through
a table such as the one below:

Proposition Reason / Justification

1 ~𝑟 Premise

2 (~𝑟) ∨ (~𝑠) 1, Rule of Addition

3 ~(𝑟 ∧ 𝑠) 2, De Morgan’s Laws

4 𝑝 → (𝑟 ∧ 𝑠) Premise

5 ~𝑝 3, 4, Modus Tollens

Note: The numbers before the rules in the reason column corresponds to the
row number of the statement from which the new statement is derived from.

Course Module
Direct Proof
An implication 𝑝 → 𝑞 can be proven by showing that if p is true, then q is also
true.
Example 2:
Give a direct proof of the theorem
“If n is odd, then n2 is odd.”
Idea: Assume that the hypothesis of this implication is true (n is odd). Then
use rules of inference and known theorems to show that q must also be true
(n2 is odd).
Proof:
n is odd.
Then n = 2k + 1, where k is an integer.
Consequently, n2 = (2k + 1)2.
= 4k2 + 4k + 1
= 2(2k2 + 2k) + 1
Since n2 can be written in this form, it is odd.
This is a direct proof.

Indirect Proof
An implication p q is equivalent to its contra-positive q p. Therefore,
we can prove p q by showing that whenever q is false, then p is also false.
Example 3:
Give an indirect proof of the theorem
“If 3n + 2 is odd, then n is odd.”
Idea: Assume that the conclusion of this implication is false (n is even). Then
use rules of inference and known theorems to show that p must also be false
(3n + 2 is even).
Proof:
n is even.
Then n = 2k, where k is an integer.
It follows that 3n + 2 = 3(2k) + 2
= 6k + 2
= 2(3k + 1)
Therefore, 3n + 2 is even.
We have shown that the contrapositive of the implication is true, so the
implication itself is also true (If 3n + 2 is odd, then n is odd).

Examples
Example 4:
General Mathematics
3
Methods of Proofs

Prove the validity of the following argument.


(𝑝 ∧ 𝑟) → (~𝑞)
(~𝑞) → 𝑟
~𝑟
∴ ~(𝑝 ∧ 𝑟)
Solution:

Proposition Reason / Justification

1 (𝑝 ∧ 𝑟) → (~𝑞) Premise

2 (~𝑞) → 𝑟 Premise

3 (𝑝 ∧ 𝑟) → 𝑟 1, 2, Syllogism

4 ~𝑟 Premise

5 ~(𝑝 ∧ 𝑟) 3, 4, Modus Tollens

Using proofs for real-life situations (Application)


Example 5:
“(1) If it rains today, then we will not have a barbeque today.
(2) If we do not have a barbeque today, then we will have a barbeque
tomorrow.
(3) Therefore, if it rains today, then we will have a barbeque tomorrow.”
First thing you need to remember is to formalize the arguments by
symbolizing the propositions.
p: “It is raining today.”
q: “We will not have a barbecue today.”
r: “We will have a barbecue tomorrow.”
(1) is symbolized by the statement: 𝑝 → 𝑞
(2) is symbolized by the statement: 𝑞 → 𝑟
(3) is symbolized by the statement: 𝑝 → 𝑟
The argument is in the form:
𝑝→𝑞
𝑞→𝑟
∴𝑝→𝑟
Thus the argument is valid.

Course Module
References
Albay, Eduard M., et al., (2016). General Mathematics. Makati City: Diwa
Learning Systems, Inc.

You might also like